ROSH: MSK/Neuro

Lakukan tugas rumah & ujian kamu dengan baik sekarang menggunakan Quizwiz!

A 64-year-old woman presents to the emergency department with fever, headache, and altered mental status. In addition to appearing toxic on examination, nuchal rigidity is demonstrated. Which other antibiotic should be considered in this patient in addition to starting ceftriaxone and vancomycin? AAmpicillin BCiprofloxacin CLinezolid DMetronidazole

Ampicillin The addition of ampicillin in patients older than 50 years of age with bacterial meningitis covers Listeria monocytogenes. Meningitis refers to inflammation or infection of the meninges that surrounds the brain and spinal cord. Infection can occur as a result of bacterial, viral, or fungal etiologies. The most common causes of bacterial meningitis include Streptococcus pneumoniae and Neisseria meningitidis. Listeria monocytogenesis a concern in patients who are immunocompromised or older than 50 years. Patients with bacterial meningitis present with fever, nuchal rigidity, headache, and altered mental status. On physical examination, the patient appears acutely ill. Later in disease, seizures, cranial nerve palsies, and papilledema may be present. Seizures and focal neurological deficits are more common in Listeria infections. Petechiae and palpable purpura may be present in Neisseria infections. On laboratory analysis, an elevated white blood cell count and thrombocytopenia may be present. Blood cultures should be obtained in all patients prior to starting antibiotic therapy. Lumbar puncture should be obtained in all patients to aid in diagnosis and help guide antibiotic therapy. A CT scan of the head should be considered prior to lumbar puncture to rule out a space-occupying lesion in patients who are immunocompromised or have a history of nervous system disease, seizures, papilledema, altered level of consciousness, or focal neurological deficits. Cerebrospinal fluid analysis reveals an elevated opening pressure, elevated white blood cell count, elevated protein level, and decreased glucose level in bacterial meningitis. Cerebrospinal fluid should be sent for culture and sensitivity. Empiric antibiotic therapyshould be initiated once blood cultures and lumbar puncture have been obtained. In healthy adult patients, vancomycin in conjunction with a third-generation cephalosporin (ceftriaxone or cefotaxime) is recommended. Dexamethasone is given in addition to reduce the risk of hearing loss and other neurological complications. Ciprofloxacin (B) is incorrect. Ciprofloxacin is used in healthcare-associated meningitis in patients who cannot take meropenem due to beta-lactam allergy. Linezolid (C) is incorrect. Linezolid is used in methicillin-resistant Staphylococcus aureus infections. It does not provide coverage for Listeria monocytogenes within the cerebrospinal fluid. Metronidazole (D) is incorrect. Metronidazole is not used in the treatment of bacterial meningitis.

Which of the following is a risk factor for Achilles tendon rupture? AAge greater than 50 years BFemale sex CFluoroquinolone antibiotic use DProfessional sports participation

Fluoroquinolone antibiotic use Achilles tendon rupture is less common in people aged greater than 50 years (A) than the peak ages of 30 to 40 years. Males have a four to five times higher rate of rupture than females (B). Professional athletes (D) have a lower incidence of tendon rupture than recreational athletes or "weekend warriors.

Which of the following physical examination findings is most likely to be present in a patient with de Quervain tendinopathy? ADifficulty actively flexing the thumb and a sensation of catching with extension BInability to make an "O" sign with thumb and index finger CPain with circumduction of the thumb carpometacarpal joint DPain with ulnar deviation of the wrist with the thumb against the palm

Pain with ulnar deviation of the wrist with the thumb against the palm De Quervain tendinopathy, also known as de Quervain stenosing tenosynovitis, affects the abductor pollicis longus and extensor pollicis brevis tendons where they pass over the radial styloid. The tendons pass through a fibro-osseous tunnel known as the first dorsal compartment. The cause of de Quervain tendinopathy is unknown but appears to be degenerative rather than inflammatory. It is often associated with repetitive movements and is more common in middle-aged women and in women during the postpartum period. Patients complain of radial sided wrist pain with movement, and the pain may radiate proximally or distally. Physical examination findings include tenderness and occasionally swelling over the tendons at the radial styloid, pain with resisted thumb extension, and pain with Finkelstein testing (ulnar deviation of the wrist with the thumb held against the palm). The diagnosis is primarily clinical, and imaging is not necessary other than to exclude other conditions, such as osteoarthritis. Treatment involves avoiding activities that exacerbate the pain, resting the tendons with a thumb spica splint, icing the hand, and taking anti-inflammatory medications. Hand therapy may be helpful for advice on activity modification and for rehabilitation as symptoms improve. Glucocorticoid injections may be used for symptoms that do not improve. Surgery to release the retinaculum over the first dorsal compartment may be indicated in refractory cases. A septum creating a separate compartment for the extensor pollicis brevis tendon is often present, so both tendons must be released.

A 29-year-old woman is evaluated in the emergency department after being bitten by a bat. The wound is cleaned and postexposure prophylaxis for rabies is indicated. Which of the following is the most appropriate next step if the patient has never received the rabies vaccine? ARabies immune globulin administered on day zero and rabies vaccine administered on days zero, three, seven, and 14 BRabies immune globulin and rabies vaccine administered on days zero, three, seven, and day 14 CRabies immune globulin only administered on days zero, three, seven, and 14 DRabies vaccine administered on day zero and rabies immune globulin administered on days zero, three, seven, and 14

Rabies immune globulin administered on day zero and rabies vaccine administered on days zero, three, seven, and 14

A 22-year-old man presents with left knee pain for one week. He states that it began during a football game when he stopped and pivoted on the knee. He had some swelling and pain for a few days after the game that eventually subsided with ice and ibuprofen. Now he feels that the knee is "catching" and he is unable to fully extend the knee without discomfort. He also feels that the knee is going to "give out" on him at times. Which of the following tests is the most sensitive for the most likely diagnosis? AAnterior drawer test BApley grind CMcMurray test DThessaly test

Thessaly test

A 23-year-old man presents to the emergency department after a head injury. Physical exam indicates eye-opening to verbal commands. When clinicians attempt to talk to the patient, he is able to engage in conversation but appears confused and disoriented. He does not obey motor commands but does localize to a painful stimulus. What is the Glasgow Coma Scale score of this patient according to these physical exam findings? A10 B12 C14 D8

12 According to these physical exam findings, this patient has a Glasgow Coma Scale (GCS) score of 12. Head injury accounts for almost half of all trauma-related deaths in young people and ranges from concussion tosevere traumatic brain injury. The grades of traumatic brain injury are defined by the GCS, which is measured 30 minutes after injury. The GCS evaluates eye-opening, verbal response, and motor response to assess the overall degree of brain injury. The minimum score is 3 and the maximum score is 15. A brain injury with a score from 13-15 is considered mild, a score of 9-12 is considered moderate, and a score ≤ 8 is considered severe. The scores are helpful in guiding treatment and diagnostic measures and also serve as prognostic indicators. Assessing ongoing scores is particularly useful for patients with epidural and subdural hematoma who may initially present with normal neurologic findings after injury but can rapidly deteriorate, indicating the need for immediate surgical intervention. In addition, current recommendations indicate that CT imaging is appropriate for any patient with a concussion who has a GCS score < 15. Patients with GCS score < 8 require intubation. Using the GCS score, this patient earns 3 points for opening his eyes to verbal command, 4 points for being conversational but disoriented, and 5 points for localizing to a painful stimulus.

A 79-year-old woman presents with a gradual onset of jaw claudication and temporal headaches. Past medical history is significant for polymyalgia rheumatica. She reports transient episodes of monocular vision loss. Physical examination reveals normal fundoscopy. Laboratory studies show an erythrocyte sedimentation rate of 129 mm/H and an elevated C-reactive protein. Which of the following is the most appropriate diagnostic test? ABiopsy BComputed tomography CMagnetic resonance imaging DPositron emission tomography

Biopsy Giant cell arteritis or temporal arteritis is an inflammatory disorder affecting medium-sized and larger vesselsthat is characterized by headaches, visual abnormalities, jaw claudication, and scalp tenderness. The mean age at onset is 79 years. Approximately 50% of patients with giant cell arteritis concurrently have symptoms of polymyalgia rheumatica, including pain and stiffness in the shoulder and pelvic girdle. Risk factors for giant cell arteritis include female sex, ethnicity, and increasing age. Giant cell arteritis tends to affect vessels with elastic tissue, including temporal, cranial, or other carotid system arteries. Patients may present with an acute or gradual onset of headaches (temporal, occipital, or frontal), scalp tenderness (especially when combing hair), visual disturbances (scotomas, blurred vision, or loss of vision), jaw claudication, and throat tenderness. Patients may also report episodes of amaurosis fugax leading to complete blindness, which is an ominous sign. Approximately 40% of patients have an atypical presentation, including aortic regurgitation, arm claudication, dry cough, mononeuritis multiplex, or fever of unknown origin. Patients older than 65 years presenting with fever of unknown origin with an elevation in acute phase reactant and normal white blood cells, giant cell arteritis should be considered. Physical examination may reveal normal or nodular, pulseless,tender, or enlarged temporal artery. Fundoscopy is commonly normal in the first 24 to 48 hours after presentation. The murmur of aortic regurgitation, asymmetry of pulses in the arm, or bruits may be present when the aorta or its branches is affected. Laboratory studies reveal a high erythrocyte sedimentation rate, typically higher than 100 mm/H. C-reactive protein may also be elevated. The complete blood count may reveal anemia, thrombocytopenia, and normal white blood cell count. Temporal artery biopsy is needed to confirm the diagnosis. However, treatment should be started immediately when giant cell arteritis is suspected. Treatment is by high-dose corticosteroids (prednisone 60 mg/day orally). Low-dose aspirin (81 mg/day) may help reduce the risk of visual loss. Patients who present with vision loss should be treated with intravenous pulse methylprednisolone 1 g/day for three days.

Which of the following tumors is most likely to cause a brachial plexus neuropathy? AAdenocarcinoma of the breast BMedullary thyroid carcinoma CNeurofibromatosis type I tumor DOsteosarcoma

Adenocarcinoma of the breast Medullary thyroid carcinoma (B) does not cause brachial plexus injury. The lesions are not in the same anatomic area of the body. Neurofibromatosis type I tumor (C) can be a rare cause of brachial plexus neuropathy. Generally, tumors of the brachial nerve sheath are uncommon and occur as solitary lesions that do not affect the functioning of the brachial plexus. However, neurofibromatosis type I can produce multitudes of tumors, and this may affect the functioning of the nerves of the brachial plexus. Nonetheless, such tumors are much rarer than breast or lung cancer. Osteosarcoma (D) is not a known cause of brachial plexus injury due to its predilection for the long bones. Of the neoplasms that can cause brachial plexopathy, lung and breast cancer are by far the most common culprits. Diagnosis of brachial plexus neuropathy is based on the history, a physical exam demonstrating decreased sensation as well as muscle weakness of the shoulder, scapula, arm or hand, and electrophysiologic studies that support neuronal loss. If the cause is traumatic or neoplastic, X-ray, MRI, or CT scan of the area may show bony deformity or tumor. MRI can show changes in the brachial nerve roots in more subtle causes. Treatment of brachial plexus neuropathy depends on the cause of the injury. Surgical repair of the brachial plexus is not always possible, especially in cases of root avulsion or severe neuronal loss. Infectious causes, such as HIV, Lyme disease, and syphilis, should be ruled out and treated if detected. Blood glucose control is paramount to halting further neuronal injury in diabetic plexopathy. Neuralgic amyotrophy generally involves pain control until the disease process runs its course. Patients with previous injury to the brachial plexus should be counseled to avoid participating in contact sports and carrying heavy backpacks.

Which of the following medications is recommended prophylactically to decrease the frequency of attacks of gouty arthritis? AAcetaminophen BAllopurinol CIbuprofen DPrednisone

Allopurinol

A 76-year-old woman presents with progressively worsening memory impairment over the past 11 months, impairing her activities of daily living. She demonstrates impaired executive functioning, anosognosia, and poor judgment. What is the most likely diagnosis? AAlzheimer disease BDementia with Lewy bodies CFrontotemporal dementia DMild cognitive impairment

Alzheimer disease

A 76-year-old woman presents with a gradual onset of forgetfulness and difficulty thinking of common words. She reports that her mother had similar symptoms when she was around the same age. The patient reports occasional difficulty recognizing her daughter's face. Her mini-mental status score is 20. Her physical examination is otherwise normal. She reports no vascular disease. Laboratory studies are normal. Which of the following is the most likely diagnosis? AAlzheimer disease BDelirium CDementia with Lewy bodies DVascular dementia

Alzheimer disease Delirium (B) is usually abrupt in onset, fluctuating in course, and has attention deficits rather than memory deficits. Dementia with bodies (C) is a form of dementia characterized by cognitive impairment, parkinsonian symptoms, well-formed visual hallucinations, and preserved short-term memory. Dementia with Lewy bodies affects more men than women, is rarely familial, and has prominent day-to-day fluctuations. Vascular dementia (D) causes an abrupt onset of neurocognitive symptoms. Patients with vascular dementia usually have a history of stroke or evidence of atherosclerotic disease. Focal neurological signs and symptoms are usually present.

Which of the following is the best preventive medication for migraine headaches? AAmitriptyline BFluoxetine CIbuprofen DRizatriptan

Amitriptyline

A 65-year-old woman with a history of breast cancer presents reporting a rash, fatigue, and difficulty climbing stairs for the past 3 months. Physical exam reveals heliotrope rash on her eyelids, erythema over her shoulders and back, and flat-topped violaceous papules over her knuckles. Which of the following lab findings is most specific for the suspected diagnosis? AAnti-centromere antibody BAnti-double-stranded DNA antibody CAnti-Jo-1 antibody DAnti-topoisomerase I antibody

Anti-Jo-1 antibody Anti-Jo-1 antibody is most specific for the diagnosis of dermatomyositis, which is described in this vignette. Dermatomyositis is a condition characterized by idiopathic myopathy with cutaneous manifestations. The disease is twice as common in women as in men and occurs in a bimodal distribution, with a smaller incidence occurring in children and a larger incidence occurring in adults aged 40-65. The cause is unknown, although it is commonly seen in association with other connective tissue disorders and is known to have an association with an occult malignancy. The classic presentation of dermatomyositis is a heliotrope rash (erythematous to violaceous patches over the eyelids), Gottron papules(flat-topped, violaceous papules over the knuckles), and shawl sign (erythema over the shoulders, upper chest, and back). Other cutaneous manifestations include periungual erythema, dilated nail bed capillaries, hyperkeratosis of the palms, and scalp involvement that mimics psoriasis. Skin involvement frequently, but not always, precedes muscle involvement. Patients present with symmetric proximal muscle weakness, most commonly affecting the shoulder and hip girdles, making it difficult to rise from a chair, climb stairs, and comb their hair. In more severe cases, patients may experience difficulty swallowing, talking, and breathing, as well as cardiac failure. Diagnosis is confirmed by muscle biopsy, which shows perifascicular atrophy. Laboratory testing reveals elevated creatine kinase and aldolase, and elevations of myositis-specific antibodies, such as anti-Jo-1 antibody and anti-Mi-2 antibody. Anti-Jo-1 antibody is most specific for the diagnosis of dermatomyositis and predicts a higher likelihood of lung involvement. Electromyography shows myopathic abnormalities including polyphasic potentials, fibrillations, and high-frequency action potentials. Treatment is with systemic steroids, but methotrexate or azathioprine can be used in patients who are resistant to or intolerant of steroids. Patients must also be reminded to avoid sun exposure, as the rash of dermatomyositis is photosensitive. A search for an underlying malignancy is also warranted in these patients.

A 34-year-old woman presents to the office for a fertility consult. She and her husband have been trying to get pregnant for over a year, and she has had four miscarriages, two in the first trimester and two during the second trimester. She has no other complaints and has not had any traumatic events or taken new medications in the past year. Physical exam is unremarkable. Laboratory testing reveals a low platelet count and high titers of several antibodies. Which of the following is the most likely diagnosis? AAntiphospholipid syndrome BGestational thrombocytopenia CPreeclampsia DSystemic lupus erythematosus

Antiphospholipid syndrome Antiphospholipid syndrome is characterized by hypercoagulability with recurrent episodes of venous orarterial thrombosis. It occurs mainly in young women in their 30s or 40s, and most patients are asymptomaticuntil a thrombotic complication occurs or with a pregnancy loss. The condition may be primary or may be associated with underlying systemic lupus erythematosus. Patients are at risk for deep venous thrombosis, pulmonary emboli, and cerebrovascular accidents. The thrombotic episodes can affect multiple organs, causing associated signs and symptoms such as thrombocytopenia, mental status change, livedo reticularis, skin ulcers, nephropathy, adrenal insufficiency, and cardiac valve dysfunction. Pregnant women with the condition may suffer unexplained fetal loss after the first trimester, multiple unexplained miscarriages during the first trimester, or premature births before 34 weeks. Diagnosis is made by testing for the presence of antiphospholipid antibodies (i.e., IgG or IgM anticardiolipin, IgG or IgM antibodies to beta-2-glycoprotein I, and lupus anticoagulant). Lifelong treatment with warfarin is recommended with maintenance of an international normalized ratio (INR) between 2.0-3.0. Those with recurrent thromboses even in this range may require a higher INR, although risks of bleeding must be weighed against benefits of anticoagulation. Pregnant women can use subcutaneous heparin and low-dose aspirin since warfarin has teratogenic effects.

Asking a patient to spell the word "world" backward during the mini-mental status exam assesses which cognitive function? AAttention BLanguage COrientation DRecall

Attention Attention is assessed when asking the patient to spell the word "world" backward. The mini-mental status exam is a 30-point questionnaire that is used to assess cognitive function. It is used extensively in both clinical and research settings and is most commonly used to screen for dementia and monitor progression and severity over time. The test evaluates various domains of cognition, including orientation, registration, attention andcalculation, recall, language, and praxis. A score ranging from 24 to 30 indicates no cognitive impairment,while a score of 18 to 23 indicates mild cognitive impairment. A score of zero to 17 indicates severe cognitive impairment. It is important to note that this test should only be used if the patient has at least an eighth-grade education and is fluent in English.

What is the genetic mode of inheritance associated with essential tremor? AAutosomal dominant inheritance pattern BAutosomal recessive inheritance pattern CX-linked dominant inheritance pattern DX-linked recessive inheritance pattern

Autosomal dominant inheritance pattern Essential tremor is associated with an autosomal dominant inheritance pattern. Essential tremor, or familial tremor, is a gradually-progressive action tremor involving the hands, head, and voice. The majority of patients will have a family history of a similar tremor due to an autosomal dominant inheritance pattern with reduced penetrance. Several different genes have been identified. Onset typically occurs on or before the age of 40 years old. Arm movements and anti-gravity positions can affect it. Patients may report that the tremor is more noticeable when writing, attempting to drink from a glass, or utilizing utensils. Tremor is more often bilateral. Anxiety and medications can aggravate tremor. Tremor is often relieved by alcohol. Progression and severity can become severe, preventing the patient from writing entirely. Secondary causes of tremor should be ruled out, such as medication side effects or thyroid disease. If the tremor is causing disability, medication should be considered to lessen the shaking. First-line treatment includes propranolol and primidone. Tolerance may be limited due to side effects of the medications. In the event of treatment failure with first-line drugs, gabapentin,topiramate, and nimodipine can be considered. Nonpharmacologic treatment options include surgical intervention, such as deep brain stimulation and thalamotomy. Botox injections can be beneficial for persistent head and vocal cord tremor

A patient presents with fluctuating muscle weakness of several weeks duration, manifesting as diplopia, ptosis, and difficulty swallowing. Physical examination reveals muscle weakness that worsens with activity and improves with rest. Which of the following is the most appropriate initial diagnostic test for this neurological condition? ABlood serum assay BCT scan CElectromyography DTensilon test

Blood serum assay Blood serum assay is the appropriate initial diagnostic test that should be performed in a patient with suspected myasthenia gravis. Myasthenia gravis manifests as inefficient skeletal muscle neuromuscular transmission due to autoimmune antibodies against the acetylcholine postsynaptic receptor. The condition is seen in all age groups but most commonly affects young women with HLA-DR3. Tensilon test (D) is specific for the diagnosis of myasthenia gravis, however, it is not performed initially. It is generally used in conjunction with electrophysiology. The administration of intravenous edrophonium, an anticholinesterase, leads to a temporary improvement of muscle strength, which can help confirm the diagnosis.

A 30-year-old man is brought to the emergency department with a gunshot to the right of his mid-back and a large laceration on his left hip. He states that his right side feels heavy and weak. He states that his left hip is not painful. On physical exam, there is a loss of vibratory sensation and proprioception of the right side. His left side has a loss of pain and temperature sensation. The patient appears to have control of his bladder. Which of the following is the most likely diagnosis? ABrown Sequard syndrome BCauda equina syndrome CConus medullaris syndrome DVentral (anterior) cord syndrome

Brown Sequard syndrome Brown Sequard syndrome is caused by injury to the spinal cord on one side. The injury damages the dorsal column, corticospinal tract, and spinothalamic tract unilaterally. Damage to the dorsal column and corticospinal tract will cause weakness, loss of proprioception, and loss of vibration on the ipsilateral side. Damage to the spinothalamic tract will cause loss of pain and temperature sensation on the contralateral side. Bladder control is preserved by the descending autonomic nerve fibers on the side that is not damaged. The most common causes of this type of lateral hemisection syndrome are knife and gunshot wounds. Diagnosis is initiated by history and physical exam followed by imaging at the level of the suspected lesion. Magnetic resonance imaging is usually the study of choice. If there is a contraindication to magnetic resonance imaging, computed tomography with myelography may be useful in diagnosing this condition.

A 74-year-old man with a history of hypertension and poor medication compliance presents to the emergency department via ambulance for loss of consciousness. Per the family, he was complaining of a severe headache and vomiting for three hours prior to the loss of consciousness. His vitals are BP 190/115 mm Hg, pulse 55 bpm, respiration 8 per minute, and O2 98% on room air. An intracerebral hemorrhage is suspected. What is the initial imaging test of choice in this patient? ACT with contrast BCT without contrast CMRI with contrast DUltrasound

CT without contrast CT with contrast (A) and MRI with contrast (C) should not be used when a bleed is suspected due to concern for the contrast pooling in the hematoma, therefore, the first-line imaging for all stroke patients is CT scan without contrast to ensure there is not a hemorrhage present before moving forward with other imaging options. An ultrasound (D) does not play a role in diagnosing an intracranial hemorrhage. Cranial ultrasounds are utilized almost exclusively in the infant population due to their open fontanelles. These ultrasounds are used to evaluate for hydrocephalus, congenital abnormalities, tumors, or intracranial hemorrhages. A cranial ultrasound would be an option if the patient in this case scenario was an infant.

A 70-year-old woman presents to the office with right knee pain for one week, which has been getting worse. On physical examination, she has an effusion in the knee, it is warm to touch, and she has pain with range of motion. Aspiration of synovial fluid reveals cloudy, yellow, nonpurulent fluid with a white blood cell count of 20,000 cells per cubic millimeter and rhomboid-shaped weakly birefringent crystals. Which of the following findings is most likely to be present on X-rays of her knee? ACalcification of the menisci BOssification of the patellar tendon COsteopenia and marginal erosions DPeriarticular soft tissue opacities and "punched-out" erosions

Calcification of the menisci The patient in the vignette has acute crystal-induced arthritis due to calcium pyrophosphate deposition disease, or pseudogout, which causes calcification of cartilage and soft tissues. Most people with calcium pyrophosphate deposition are asymptomatic, and the condition is discovered incidentally on X-rays. It most commonly affects the knee, along with the wrist, elbow, shoulder, and hip, but can affect any joint. Calcifications may be seen in the knee menisci, the symphysis pubis, the triangular fibrocartilage of the wrist, and the glenoid and acetabular labra. However, a patient may present with an acute attack, which is similar to an attack of gout, with severe pain, warmth, and swelling of a joint. The knee is most commonly involved in pseudogout, as opposed to the first metatarsophalangeal joint in gout. Calcium pyrophosphate deposition disease can also present as chronic arthritis, similar to rheumatoid arthritis. X-rays showing calcification of the menisci support the diagnosis of pseudogout. Aspiration of synovial fluid reveals a white blood cell count in the low inflammatory range, typically 15,000 to 30,000 cells per cubic millimeter, and calcium pyrophosphate crystals, which are rhomboid-shaped and weakly positively birefringent, compared with uric acid crystals found in gout, which are needle-shaped and strongly negatively birefringent. Treatment for an acute pseudogout attack includes nonsteroidal anti-inflammatory medications, colchicine, or systemic glucocorticoids for patients who can not tolerate either. Aspiration of joint fluid along with intra-articular glucocorticoid injection is also beneficial. Colchicine may be used as a prophylactic medication for patients with frequent multiple attacks.

A 72-year-old woman with type II diabetes presents to your office with swelling in her left foot, which she noticed five days ago. She does not have any history of injury and has minimal pain. On physical examination, she has diffuse warmth, erythema, and edema over her foot. There is collapse of the longitudinal arch and prominence of the navicular. Her serum white blood cell count, erythrocyte sedimentation rate, and C-reactive protein are normal. Which of the following is the most likely diagnosis? ACharcot foot BGout COsteomyelitis DPosterior tibial tendon dysfunction

Charcot foot The patient in the vignette has findings consistent with a Charcot foot or diabetic neuropathic arthropathy. This condition is characterized as the chronic, progressive destruction of joints due to loss of sensation. It was originally described in tabes dorsalis, but currently, the most common cause is diabetic neuropathy. The pathophysiology is not completely understood but appears to be related to loss of proprioception and pain sensation, which predisposes the foot to minor trauma and altered weight-bearing patterns. Vasomotor dysfunction from autonomic neuropathy and localized inflammation may lead to increased bone resorption and formation. Early in the course of the disease, physical examination findings include swelling, warmth, and erythema with minimal pain, and X-rays are normal. Later, X-ray changes occur with fractures, joint subluxations, osteolysis, bone destruction, and bony debris. The X-ray abnormalities may progress rapidlyand can be confused with traumatic injuries, stress fractures, or osteomyelitis. The midfoot and hindfoot are most commonly involved. Eventually, the swelling and warmth resolve and the bone remodels. X-rays during this phase may show signs of fracture healing and new bone formation. Advanced imaging, such as technetium bone scanning and magnetic resonance imaging, may be necessary if the diagnosis is in doubt. Tagged white blood cell scanning may be needed to differentiate neuropathic arthropathy from infection. Treatment is offloading the affected foot. Many patients have difficulty avoiding weight-bearing, so total contact casting is the preferred treatment. The cast should be changed frequently, since it will become loose as the edema subsides. Patients may need prolonged periods of casting, from several months to two years, followed by bracing. Once bone healing is complete, patients can transition to weight-bearing in normal shoes. Other treatments that have been attempted include bisphosphonates and calcitonin, but studies on these medications have been inconclusive. Reconstructive surgery may be needed for severe deformity once the inflammatory changes have subsided, however, surgery has a high rate of complications, including amputation.

A 37-year-old woman presents to her primary care provider with complaints of left hip pain. She does not recall any recent injuries. A review of her current medications is concerning for avascular necrosis. Which of the following medications is associated with development of this problem? ABisphosphonates BFluoroquinolone antibiotics CGlucocorticoids DStatins

Glucocorticoids Bisphosphonates (A) decrease the rate of bone resorption and are one of the medications used in the non-surgical treatment of osteonecrosis. Fluoroquinolone antibiotics (B) are a medication associated with tendon rupture but not with avascular necrosis. Statins (D), commonly prescribed to treat hyperlipidemia, are associated with muscle toxicity but are thought to prevent osteonecrosis and may be part of a non-surgical treatment plan.

Which gastrointestinal pathogen is classically associated with reactive arthritis? ACampylobacter jejuni BChlamydia trachomatis CStaphylococcus aureus DStreptococcus pneumoniae

Campylobacter jejuni Campylobacter jejuni is the gastrointestinal pathogen classically associated with reactive arthritis. Reactive arthritis, formerly known as Reiter Syndrome, is an inflammatory arthritis that arises following a genitourinary or gastrointestinal infection. It is generally regarded as a form of spondyloarthritis (SpA), which is a term referring to inflammatory diseases that involve both the joints and the entheses. The entheses are the sites where ligaments or tendons attach to bones. Other examples of spondyloarthritis conditions are ankylosing spondylitis and psoriatic arthritis. The most common pathogens associated with reactive arthritis are Chlamydia trachomatis, Campylobacter jejuni, Salmonella, Shigella, Escherichia coli, and Clostridium difficile. Chlamydia trachomatis is a common genitourinary pathogen, whereas the other pathogens commonly associated with reactive arthritis are usually enteric pathogens. In reactive arthritis, the associated pathogens cannot be cultured from the affected joint, and the timing between the preceding infection and the arthritis can range from days to weeks. Reactive arthritis is most commonly seen in young men, with greater male predominance following genitourinary infection than enteric infection. Symptoms typically present as asymmetric inflammatory oligoarthritis that often involves the lower extremities and is associated with enthesitis. The knees and ankles are most commonly affected, however, sacroiliitis may be observed less commonly. Extra-articular manifestations are common and include urethritis, conjunctivitis, uveitis, and mucocutaneous lesions. Fever and weight loss are common at the onset of disease. Mucocutaneous lesions include balanitis, stomatitis, and keratoderma blennorrrhagicum of the soles of the feet. The diagnosis of reactive arthritis is a clinical diagnosis based on the exclusion of other diseases. Evidence of an antecedent or concurrent infection is important. Acute phase reactants, such as erythrocyte sedimentation rate (ESR) or C-reactive protein (CRP), are elevated in less than 50% of cases of reactive arthritis. Patients with reactive arthritis are more likely to have a positive human leukocyte antigen (HLA)B27 than the general population. This test is sometimes ordered to support the diagnosis. Reactive arthritis is usually self-limited. If the patient has a current Chlamydia trachomatis infection, it should be treated with antibiotics. The acute arthritis symptoms seen with reactive arthritis should be treated with nonsteroidal anti-inflammatory drugs (NSAIDs). For patients who do not respond to NSAIDs, glucocorticoids, sulfasalazine, methotrexate, or anti-TNF agents may be prescribed.

Which of the following pathogens is most commonly associated with Guillain-Barré syndrome? ACampylobacter jejuni BCytomegalovirus CEpstein-Barr virus DMycoplasma pneumoniae

Campylobacter jejuni Campylobacter jejuni is the most common pathogen implicated in the development of Guillain-Barré syndrome. Guillain-Barré syndrome is an acute and subacute demyelinating polyradiculoneuropathy that results insymmetric ascending weakness and sensory impairment. Although the exact cause of Guillain-Barré syndrome is unknown, it is thought to have an immunologic basis. The condition commonly follows a gastrointestinal or respiratory illness by two to four weeks and has also been found to occur following immunizations or surgery. Campylobacter jejuni is the most commonly implicated organism, however, other preceding infections include cytomegalovirus, Epstein-Barr virus, Mycoplasma pneumoniae, and varicella-zoster virus, among others. Symptoms vary widely between patients, but most commonly, patients present withweakness in the legs that spreads proximally to the arms and face. Respiration and deglutition may also be affected. Patients may also experience sensory disturbances, such as distal paresthesias, although these symptoms tend to be less pronounced than the motor symptoms. Autonomic symptoms may also occur, including tachycardia, facial flushing, hypotension, hypertension, and cardiac irregularities. Physical examination usually reveals symmetric lower extremity weakness with diminished or absent reflexes. Diagnosis is usually made clinically, but laboratory testing, electromyography, and nerve conduction studies can be performed to confirm diagnosis. Cerebrospinal fluid analysis shows elevated protein with normal WBC cell count, termed albuminocytological dissociation, which is highly supportive of the diagnosis. While results of electromyography and nerve conduction studies can assist in confirming the diagnosis, it is important to note that the tests may be normal in a patient with Guillain-Barré syndrome. Pulmonary function tests should be conducted frequently to assess for declining respiratory function and the need for assisted ventilation. Treatment includesplasmapheresis, which is most effective in the first few days of illness and for rapidly progressive cases where ventilation is required. Intravenous immunoglobulin has also been shown to be equally effective. All patients with declining respiratory function should be admitted to the intensive care unit for respiratory support. The majority of patients with Guillain-Barré syndrome make a full recovery, however, up to 20% of patients are left with permanent disability.

Which of the following is the treatment of choice for trigeminal neuralgia? AAmitriptyline BCarbamazepine CIbuprofen DPrednisone

Carbamazepine The trigeminal nerve supplies sensation to the face and is responsible for both sensory and motor supply to the muscles of mastication. The nerve has three main divisions: ophthalmic (V1), maxillary (V2), and mandibular (V3). Trigeminal neuralgia is defined clinically by a sudden onset of severe and usually unilateralepisodes of stabbing pain in the distribution of one or more branches of the fifth (trigeminal) cranial nerve. The sharp and stabbing episodes usually last one to several seconds and is near maximal in intensity at the onset. These episodes can occur repetitively with a refractory period of several minutes between each episode. Trigeminal neuralgia paroxysms can be triggered by chewing, talking, brushing teeth, cold air, smiling, and grimacing. Trigeminal neuralgia can be subclassified as classic trigeminal neuralgia, secondary trigeminal neuralgia, or idiopathic trigeminal neuralgia. Classic trigeminal neuralgia consists of cases related to vascular compression of the trigeminal nerve. Secondary trigeminal neuralgia is caused by an underlying disease, such as multiple sclerosis or compression of the trigeminal nerve by a space-occupying lesion. Characteristic clinical features clinically confirm the diagnosis of trigeminal neuralgia. All patients with suspected trigeminal neuralgia should undergo neuroimaging to help distinguish classic from secondary trigeminal neuralgia. Magnetic resonance imaging (MRI) is the preferred imaging modality. The initial treatment for trigeminal neuralgia is pharmacologic therapy with carbamazepine. For patients that do not respond to carbamazepine, oxcarbazepine may be used as a second-line treatment. Alternatively, baclofen with or without the addition of lamotrigine may be used in patients who do not have adequate pain relief with carbamazepine and oxcarbazepine. In addition, treatment of the underlying condition causing secondary trigeminal neuralgia is recommended. Patients who are refractory to medical therapy may benefit from surgical therapy using microvascular decompression, rhizotomy, or gamma knife surgery. In general, patients with trigeminal neuralgia have a high rate of spontaneous remission, although relapses are common.

A 45-year-old man presents to the emergency department with a spine injury after a motorcycle accident. He is alert and oriented and complains of severe pain in his lower back and both legs. On physical examination, he has decreased sensation over the perineum, weakness of plantar flexion and absent ankle jerk reflexes bilaterally, and fullness and tenderness in the suprapubic area. Which of the following is the most likely diagnosis? ACauda equina syndrome BCentral cord syndrome CDisc herniation at the L5-S1 level DSpinal stenosis

Cauda equina syndrome The cauda equina (Latin for "horse's tail") is the bundle of nerve roots below the level of the spinal cord. The spinal cord usually ends at the level of the L1 or L2 vertebrae but can end anywhere between T12 and L3. Cauda equina syndrome is compression of the nerve roots below the distal end of the spinal cord. Patients present with symptoms of lower back and leg pain, often severe; weakness and absent reflexes in the lower extremities; loss of sensation in a dermatomal distribution; saddle anesthesia; and urinary incontinence due to urinary retention and overflow incontinence. Physical examination shows decreased reflexes, sensory deficits, and decreased rectal tone (no "anal wink"). Magnetic resonance imaging or CT myelogram are indicated immediately, as cauda equina syndrome is a surgical emergency. Surgical decompression of the nerves should be performed within 24 to 48 hours. Studies have shown better return of bowel and bladder function and motor and sensory function when surgery is done within 48 hours of symptom onset.

Which of the following diagnostic studies is used in conjunction with electrodiagnostic evaluation to aid in the diagnosis of Guillain-Barré syndrome? AAntibody testing BCerebrospinal fluid analysis CEvoked potentials DMagnetic resonance imaging

Cerebrospinal fluid analysis Antibody testing (A) is incorrect. Antibody testing can be helpful in diagnosing Miller Fisher variant of Guillain-Barré. Evoked potentials (C) is incorrect. This is a diagnostic study more often used in multiple sclerosis. Magnetic resonance imaging (D) is incorrect. MRI can show enhancement of cauda equina and descending nerve roots but is not necessarily needed for making the diagnosis.

A 40-year-old man is in the emergency department with a head injury following a motorcycle accident. While waiting to be admitted, the patient begins to shake. There are bilateral tonic and clonic motor activities, and the patient loses consciousness. The rhythmic jerking lasts three minutes then subsides. It resumes two minutes later with no return of consciousness. The pattern repeats two more times without the return of consciousness. Which of the following would be considered diagnostic for the condition? AClinical diagnosis BComputed tomography CElectroencephalography DMagnetic resonance imaging

Clinical diagnosis Status epilepticus is a neurologic emergency requiring rapid evaluation and intervention to prevent neuronal injury, neuronal alteration, or neuronal death. Status epilepticus is defined as five minutes or more of continuous seizure activity or more than one seizure with no return of consciousness in between seizures. While epilepsy, especially if medication compliance is not adhered to, is a common etiology in adults, there are other causes of the condition. Brain injuries, such as stroke, trauma, or subarachnoid hemorrhage, can lead to status epilepticus. Withdrawal from alcohol, barbiturates, or benzodiazepines or overdose or use of drugs that lower the seizure threshold can also lead to this condition. Metabolic abnormalities and infection have also been noted to cause status epilepticus. In children, the most common cause of status epilepticus is febrile seizure. In addition to multiple etiologies, there are a few different presentations depending on the type of seizure. The most serious is the generalized convulsive status epilepticus. It involves both tonic and clonic movements and a loss of consciousness. Focal motor status epilepticus may or may not have loss of consciousness. The area affected by the seizure is limited to a single limb or one half of the body, depending on the location of the trauma to the brain. Jacksonian march is a type of focal motor seizure. Myoclonic seizures can be either generalized or focal and may be more benign than other types. Diagnosis is clinical. The condition should be treated without waiting for imaging. Initial treatment is with a benzodiazepine, usually intravenous lorazepam. Midazolam can be used if intravenous access is not available. Long-term therapy is with an antiseizure medication from a non benzodiazepine class.

A 32-year-old man with suspected carbon monoxide poisoning is evaluated in the emergency department. He is unarousable and unresponsive. Which of the following terms accurately describes this patient's condition? AComatose BLethargic CObtunded DStuporous

Comatose Comatose is the term used to describe a state of unarousable unresponsiveness. Patients who are comatose have an impaired response (or entire lack of response) to external stimuli and are difficult to arouse (or are entirely unarousable). An alteration of responsiveness and arousal can be life-threatening. Differential diagnosis is broad, so the pathophysiology can differ. Damage to the ascending reticular activating system within the upper brainstem or to the bilateral diffuse cerebral hemisphere can result in altered awareness. In addition to trauma and seizures, coma can result from toxic, metabolic, and infectious etiologies. Obtaining an accurate history in a comatose patient is imperative but difficult given that history cannot be physically obtained from the patient. Witnesses, family members, and any first responders who may have been present can provide information on medication history, alcohol and drug use, prior medical history, the last time the patient was seen normal, traumatic injury, and any other additional information. A detailed physical and neurological examination should be performed with particular attention given to motor responses and brainstem reflexes. Motor examination evaluates for muscle tone, hyperreflexia, spontaneous or purposeful movements, and abnormal posturing. Brainstem reflexes include pupillary light reflex, eye movements (oculocephalic maneuver or caloric testing), and corneal reflex. Glasgow Coma Scale score should be determined, which is based on eye opening, verbal response, and motor response on a scale of 3-15. Comatose patients should be urgently evaluated with testing based on their clinical presentation. Laboratory testing, lumbar puncture, electroencephalography, and neuroimaging may be indicated. Patients with a Glasgow Coma Score of 8 or less should be intubated. Pharmacologic therapy varies depending on the etiology of coma. Dextrose is recommended while waiting for blood test results if cause of coma is unknown, however, thiamine should be given first if Wernicke encephalopathy is suspected. Prognosis is dependent on the underlying cause.

patient presents to the emergency department following a seizure. Witnesses report that they saw the patient staring into space followed by repetitive lip smacking and chewing motions that lasted about two minutes. During this period of time, the patient was not responsive to external stimulation and remained confused for several minutes after the episode. Which of the following describes the type of seizure this patient experienced? AAbsence seizure BComplex partial seizure CSimple partial seizure DTonic-clonic seizure

Complex partial seizure Complex partial seizure is described in this vignette. Seizures are transient disturbances in cerebral function due to an abnormal paroxysmal neuronal discharge in the brain. Seizures can be classified as generalized when they affect the brain diffusely and partial, or focal, when they affect only a restricted part of the brain. Because partial seizures only involve one part of the brain, the ictal manifestations reflect the area of the brain that is affected. A complex partial seizure is characterized by isolated focal tonic or clonic activity, repetitive movements, and sensory disturbances that occur in the setting of altered consciousness lasting fromseconds to minutes. The patient often does not recall the seizure occurring. These repetitive movements,known as automatisms, are characteristic of complex partial seizures and can present as lip smacking, chewing movements, and repetitive picking at their clothes. Frequently these patients experience an aura prior to seizure onset, in the form of altered sensation, perception, memory, or emotion. Diagnosis is made using clinical and diagnostic studies. In a patient with an initial presentation of a focal seizure, MRI should be performed to rule out an underlying structural abnormality. Routine lab monitoring, including complete blood count, serum glucose, electrolytes, creatinine, calcium, magnesium, liver function tests, should be performed initially to rule out any underlying disorder. If infection is suspected, a lumbar puncture should be performed but only after appropriate imaging if focal signs are suspected. Electroencephalography (EEG) in a complex partial seizure often reveals interictal spikes or spikes associated with slow waves in the temporal or frontotemporal areas. Occasionally, however, no ictal activity will be seen. Treatment with antiepileptic medications is initiated once a patient presents with more than one unprovoked seizure and is continued until the patient is seizure free for at least two years. For a patient with only one unprovoked seizure, treatment with medication is not warranted unless there is a high risk of recurrence. Carbamazepine is considered the standard first-line treatment for focal seizures, although phenobarbital, primidone, phenytoin, and valproic acid may also be used. The newer anticonvulsants, such as gabapentin, topiramate, lamotrigine, oxcarbazepine, and levetiracetam, are considered second-line, although many neurologists may now use these medications first in clinical practice.

Which of the following cerebrospinal fluid analysis findings is most supportive of bacterial meningitis? ADecreased glucose concentration BElevated lymphocyte count CNormal protein concentration DNormal white blood cell concentration

Decreased glucose concentration Community-acquired bacterial meningitis is a condition defined by inflammation of the meninges that is due to a bacterial infection acquired in the community. The most common pathogens vary according to the age of the patient. In neonates less than 1 month of age, the most common pathogens are Escherichia coli, Group BStreptococcus, and Listeria monocytogenes. In patients two to 50 years old, the most common pathogens are Streptococcus pneumoniae (pneumococcal meningitis) and Neisseria meningitidis (meningococcalmeningitis). In adults older than 50 years of age, the most common pathogens are Streptococcus pneumoniae, Neisseria meningitidis, and Listeria monocytogenes. Risk factors to developing meningitis include chronic medical conditions (diabetes mellitus), immunocompromised states, crowded living conditions (dormitories), injection drug use, recent head trauma, recent travel, alcoholism, otorrhea or rhinorrhea, contact with individuals known to have meningitis, prolonged steroid use, and asplenia. The classic symptoms include fever, headache, altered mental status, and neck stiffness. Although many patients with bacterial meningitis will not have all of these findings, the majority of patients will have at least two. Classic physical exam findings include nuchal rigidity, Brudzinski sign, and Kernig sign. Brudzinski sign refers to spontaneous flexion of the hips during passive flexion of the neck. Kernig sign refers to resistance to full extension of a patients knee when the ipsilateral hip is flexed to 90. Patients with suspected bacterial meningitis require blood culturesand a lumbar puncture with cerebrospinal fluid analysis to confirm the diagnosis. However, some patients require a computed tomography (CT) scan of the brain prior to obtaining a lumbar puncture if signs ofincreased intracranial pressure are present to avoid brain herniation. Indications for a computed tomography scan include immunocompromised status, new-onset of seizures within the past seven days, a prior history of central nervous system disease (e.g., stroke, mass lesion, meningitis, encephalitis), reduced level of consciousness (e.g., lethargy, comatose), papilledema, and focal neurologic deficits. Since most patients will not have any of these indications, a computed tomography of the head is usually not necessary. Patients who do not require a computed tomography should have blood cultures and a lumbar puncture forcerebrospinal fluid analysis obtained prior to initiating dexamethasone and antibacterial therapy. The diagnosis of bacterial meningitis is confirmed with the results of the cerebrospinal fluid analysis, which should include a white blood cell count with differential, protein concentration, glucose concentration, lactic acid, Gram stain, and culture. The classic findings on cerebrospinal fluid analysis include an elevated white blood cell count, elevated protein count, and a decreased glucose concentration. Lactic acid is used to distinguish between viral and bacterial causes of meningitis. The antibiotic regimen should be targeted to the Gram stainand culture as the results are obtained. Dexamethasone is started empirically in patients with bacterial meningitis but should only be continued if the culture indicates Streptococcus pneumoniae as the causative bacteria. The empiric treatment for community-acquired bacterial meningitis in individuals who are 2 months to 50 years of age should include vancomycin and ceftriaxone. Individuals older than 50 years of age should be treated with vancomycin, ceftriaxone, and ampicillin. The mortality rate of bacterial meningitis increaseswith increasing age. Neurologic complications often occur in individuals with bacterial meningitis. Neurologic complications may include impaired mental status, seizures, focal neurologic deficits (e.g., cranial nerve palsy or hemiparesis), sensorineural hearing loss, and intellectual impairment. Vaccinations are available to reduce the risk of meningitis. These include the quadrivalent meningococcal conjugate vaccine and the serogroup B meningococcal vaccine. Both vaccines are recommended for healthy adolescents and young adults. In addition, chemoprophylaxis is indicated in individuals who had close contact with patients with meningococcal infection and should be given as soon as possible after exposure. Close contact is generally defined as anyone who has had prolonged (more than eight hours) contact in close proximity (within three feet) to the patient or anyone who has been directly exposed to the individuals oral secretions during the seven days prior to the onset of the patients symptoms. The recommended agents used for chemoprophylaxis include rifampin, ciprofloxacin, or ceftriaxone.

A 43-year-old man with a history of back pain is found to have a herniated disc of the lumbar spine at the level of L5. Physical exam reveals 4-5 strength and shows diminished sensation. Which of the following best describes the location of diminished sensation in this patient? ADorsal foot BLateral foot CMedial ankle DPosterolateral thigh

Dorsal foot

Which of the following is the most appropriate treatment for an acute, closed, soft-tissue mallet injury of a finger? ADorsal hyperextension splinting of the distal interphalangeal joint for six weeks BOpen repair of the terminal extensor digitorum tendon CPercutaneous K-wire fixation of the distal interphalangeal joint in extension for six weeks DReconstruction of the terminal extensor digitorum tendon using a tendon graft

Dorsal hyperextension splinting of the distal interphalangeal joint for six weeks

Which of the following clinical manifestations is most indicative of trigeminal neuralgia? AConstant dull aching facial pain BEpisodes of sudden lancinating facial pain worsened with touch CLancinating pain that starts near the eye and ear and shoots to the mouth DUnilateral facial paresis, hyperacusis, and impaired taste

Episodes of sudden lancinating facial pain worsened with touch Episodes of sudden lancinating facial pain worsened with touch is most characteristic of trigeminal neuralgia. The exact cause of trigeminal neuralgia is unknown but is thought to be due to an anomalous artery or vein impinging on the trigeminal nerve (CN V), most commonly in the second and third divisions of the nerve. It most commonly affects women and has an onset in middle and later life. Patients with trigeminal neuralgia present with sudden, lancinating facial pain that starts on one side of the mouth and shoots towards the ear, eye, or nostril on the ipsilateral side of the face. Pain is worsened by touch, movements, eating, and drafts of wind. Patients may hold their face still while talking to lessen severity of pain. There are often periods of remission, lasting weeks to months, however, as the disorder progresses, episodes of pain become more frequent. Diagnosis is made clinically and neurological exam is normal. Multiple sclerosis should be suspected in patients younger than 40 even when trigeminal neuralgia is the only symptom, as it may be one of the first symptoms. In cases where secondary causes are suspected or symptoms are bilateral, MRI is warranted. First-line treatment is with carbamazepine, which should be prescribed with regular monitoring of serial blood counts and liver function tests. In patients not responsive to carbamazepine, phenytoin, lamotrigine, and gabapentin may be prescribed. For patients who do not improve with pharmacologic therapy, surgical decompression of the trigeminal nerve may offer symptomatic relief.

A 27-year-old man with Lyme disease presents to the emergency department due to sudden onset of inability to move the right side of his face. He is unable to raise his right eyebrow or close his right eye, he has drooping of the right side of his mouth, and loss of the right nasolabial fold. His examination suggests involvement of what cranial nerve? AAbducens nerve BBells nerve CFacial nerve DTrigeminal nerve

Facial nerve

A 75-year-old man presents to the emergency department with severe right knee pain that began yesterday. Examination shows a large effusion in the knee, warmth, erythema, diffuse tenderness to palpation, and pain with range of motion. Laboratory tests show a normal white blood cell count, normal uric acid level, and erythrocyte sedimentation rate of 50 mm/hr. Synovial fluid analysis yields a white blood cell count of 50,000 cells per cubic millimeter and needle-shaped, negatively birefringent crystals. Which of the following is the most likely diagnosis? ACalcium pyrophosphate deposition disease BGout CReactive arthritis DSeptic arthritis

Gout

A 55-year-old woman presents with a gradual onset of a cough, dyspnea, and red eyes for 11 months. She has been treated multiple times for otitis media and sinusitis. She also reports ankle pain that has migrated to her knee. Physical exam reveals saddle nose deformity, nasal septum crusting, anterior uveitis, and otitis media. Laboratory tests reveal a positive antineutrophil cytoplasmic antibody directed towards proteinase-3. Anticyclic citrullinated peptide antibody and rheumatoid factor are absent. Tissue biopsy shows granulomatous inflammation and geographic necrosis. Which of the following is the most likely diagnosis? AGranulomatosis with polyangiitis BMicroscopic polyangiitis CRefractory sinusitis DRheumatoid arthritis

Granulomatosis with polyangiitis Granulomatosis with polyangiitis, formerly known as Wegener granulomatosis, is an antineutrophil cytoplasmic antibody disorder characterized by the triad of glomerulonephritis and upper and lower respiratory tract disease. It is associated with vasculitis of small vessels including arteries, arterioles, and capillaries. The incidence of granulomatosis with polyangiitis is 12 per 1,000,000 per year. Men and women are equally affected predominantly in the fourth and fifth decades of life. Patients present with gradual onset (4-12 months) of upper and lower respiratory tract symptoms, including nasal congestion, otitis media, sinusitis, mastoiditis, and gingivitis, that are refractory to conventional treatment. Patients with advanced disease may present with a cough, dyspnea, and hemoptysis. Signs of glomerulonephritis including hypertension, edema,and elevated creatinine level may be present and rapidly progresses if treatment is delayed. Other symptoms of granulomatosis with polyangiitis may include scleritis, oligoarthritis of the large joints, episcleritis, anterior uveitis, skin lesions or purpura, fever, weight loss, and malaise. Physical exam findings may include saddle nose deformity, proptosis, and deep vein thrombosis. Laboratory studies may reveal positive antineutrophil cytoplasmic antibody directed towards myeloperoxidase or proteinase-3 (90% specificity). Urinalysis usually reveals red cell sediments, red cell casts with or without white cell cast. Tissue biopsy remains the best confirmatory test and shows granulomatous inflammation and geographic necrosis. Computed tomography of the chest may show granulomas, infiltrates, nodules, or masses. Treatment is twofold and includes inducing and maintaining remission. Disease severity can be determined using the Birmingham vasculitis activity score, a scoring system that incorporate data on disease manifestation in nine categories (arthralgia, arthritis, and fever plus involvement of eight organ systems). Mild disease is defined as non-organ-threatening and non-life-threatening while severe disease is defined as life-threatening or organ-threatening. Inducing remission in mild disease may include the administration of glucocorticoids and methotrexate. In severe disease, inducing remission may be achieved by combining corticosteroids with either rituximab or cyclophosphamide. Once remission is induced, maintenance therapy, using either azathioprine or methotrexate, should be initiated.

Which of the following best describes a deformity of a lesser toe in which the metatarsophalangeal joint is extended and the proximal interphalangeal joint is fixed in flexion? AClaw toe BCurly toe CHammertoe DMallet toe

Hammertoe

A 32-year-old woman presents with low back pain for the last two days. She states that it began gradually after carrying several large boxes upstairs. She has not tried anything to alleviate the pain. She denies paresthesias, numbness, and weakness. She denies any radiation of the pain into the lower extremities. Physical exam shows limited range of motion of the lumbar spine in flexion at endpoints, tenderness in the right paraspinal musculature without any midline tenderness, and normal heel and toe walking. Which of the following treatment plans would be most appropriate to implement at this time? ABed rest BHeat therapy CMassage therapy DPhysical therapy

Heat therapy Acute low back pain is common and associated with the following risk factors: sedentary job, female gender, smoking, obesity, advanced age, and depression. The majority of patients who present with back pain are diagnosed with nonspecific back pain. This is defined as back pain in the absence of a pathologic cause and is often associated with musculoskeletal strain. Serious etiologies must be ruled out with a detailed history and physical examination. History should include questions about changes in sensation, numbness, or weakness. Questions about fever, night sweats, or unintentional weight loss can help differentiate malignancy as the cause. Questions about intravenous drug use or recent infection can help rule out epidural abscess. Physical examination should focus on ruling out causes of back pain that warrant further diagnostics or imaging. Neurologic exam should include reflexes, strength, gait, and sensation of the lower extremities. Palpation of the midline that reveals tenderness may be a sign of vertebral metastases, compression fracture, or spinal infection. The straight leg raise will elicit pain associated with lumbar radiculopathy. Treatment options include nonpharmacologic and pharmacologic options. If patients wish to try nonpharmacologic options, heat therapy has been shown in studies to reduce muscle spasm. Other therapies including massage, acupuncture, and self-care exercises have not been shown to be as efficacious, however, patients may find some relief in these modalities.First-line pharmacologic therapy includes nonsteroidal anti-inflammatory drugs (NSAIDs). Acetaminophen has not been shown to be as helpful in alleviating pain. Second-line treatment includes a nonbenzodiazepine muscle relaxant. Prognosis is good with most cases resolving within seven weeks of onset.

Which of the following medications is the first-line treatment for giant cell arteritis? AHigh-dose prednisone BLow-dose prednisone CMethotrexate DNonsteroidal anti-inflammatory drugs

High-dose prednisone High-dose prednisone is the first-line treatment for giant cell arteritis. Giant cell arteritis is a systemic vasculitis affecting medium and large sized vessels. It is frequently called temporal arteritis, as it frequently affects the temporal artery in addition to other extracranial branches of the carotid artery. Giant cell arteritis frequently coexists with polymyalgia rheumatica, which is characterized by pain and stiffness in the proximal muscles that lasts several weeks and is not caused by underlying conditions. Both giant cell arteritis and polymyalgia rheumatica occur in adults over 50, with the mean age being 79 years, and occurs most commonly in women. The cause of giant cell arteritis is unknown but is thought to be associated with cell-mediated immunity. There is a suspected association with varicella-zoster antigen, as up to 75% of patients with giant cell arteritis are found to have temporal artery antigens compared to controls. Symptoms of giant cell arteritis include unilateral temporal headache, scalp tenderness, jaw claudication, or throat pain. In addition, patients may experience diplopia, scotomas, or permanent vision loss. It is important to note that fundoscopic findings may not be seen for the first 24-48 hours of vision changes. Physical exam may reveal a normal appearing temporal artery, however, patients may also present with a nodular, enlarged, tender, or pulseless temporal artery. Patients with involvement of the aorta and its branches will demonstrate asymmetrical pulses in the arms, aortic regurgitation murmurs, and bruits over the clavicle. Older patients may not present with classic symptoms and may only exhibit a fever, thus there should be a high index of suspicion for this condition. Diagnosis is made clinically and is confirmed with temporal artery biopsy. Lab findings support the diagnosis and show ESR levels over 50 mm/h in up to 90% of patients. C-reactive protein levels are also shown to be elevated in most patients, however, normal values cannot exclude the diagnosis. Because permanent blindness is the most devastating complication if not immediately treated, patients must be treated first, before temporal artery biopsy is performed for confirmation. Treatment is with immediate high-dose prednisone, usually 60 mg/day orally that is continued for one month before tapering. Most patients with giant cell arteritis will stay on a lower dose of prednisone for an average of two years.

A 65-year-old man is brought in by ambulance after being found unresponsive. A CT scan of the head demonstrates an acute hemorrhage in the left putamen. Which of the following is the most common risk factor of hemorrhagic stroke? AAntiplatelet therapy BCocaine use CHypertension DTrauma

Hypertension Hypertension is the most common risk factor for hemorrhagic stroke. Intracerebral hemorrhage is the second leading cause of stroke after infarction. Besides elevated blood pressure, other risk factors include cerebral amyloid angiopathy, vascular malformation, aneurysm rupture, trauma, excessive alcohol use, and anticoagulation therapy. Patients will often present with decreased level of consciousness, headache, confusion, hemiplegia, and visual disturbance. The most common areas of the brain involved are the putamen, internal capsule, and thalamus. Emergent evaluation with imaging of the head is of utmost importance. Noncontrast CT scan of the head is favored as initial test due to its immediate availability. Magnetic resonance imaging of the brain can also be performed. Coagulation studies should be obtained. Patients with hemorrhagic stroke will often be transferred to the intensive care unit for aggressive blood pressure control (depending on systolic) and mechanical ventilation. Anticoagulation may be reversed in appropriate individuals. Patients often need intracranial pressure monitoring. Some individuals may be candidates for ventriculostomy and evacuation of the hematoma. Complications of hemorrhagic stroke can include permanent paralysis and death.

A 12-year-old girl presents to the office with a three-month history of pain in both knees that is worse on the left. She plays soccer and runs cross-country on her middle school teams. The pain is worse with activity and helped by rest, but it has worsened to the point where she has had to sit out of practices and games. On physical examination, she has no effusion in either knee. She has mild swelling and tenderness of the tibial tubercles bilaterally. She has full range of motion of the knees and has pain with resisted knee extension. Which of the following is the most appropriate treatment? AAvoidance of sports until completely asymptomatic BIce, acetaminophen or ibuprofen, and gradual return to activity CKnee immobilizer and crutches for four weeks DTherapeutic ultrasound

Ice, acetaminophen or ibuprofen, and gradual return to activity

A 76-year-old man with a history of chronic alcohol use is brought to the emergency department by a family member with a report of acutely altered mental status. In addition to marked gait ataxia, which of the physical exam findings below would be consistent with a diagnosis of Wernicke encephalopathy? AAsterixis BCaput medusa CLateral rectus palsy DPtosis

Lateral rectus palsy Asterixis (A) is incorrect. Asterixis is associated with hepatic encephalopathy. Caput medusa (B) is incorrect. While it may be seen in individuals with a history of chronic alcohol use and cirrhosis, caput medusa is not associated with Wernicke encephalopathy. Ptosis (D) is incorrect. Ptosis is not a common ocular finding in patients with Wernicke encephalopathy.

A 7-year-old boy presents to the office with his mother, who states that he has been limping on and off for the past three months. Occasionally, he complains of right knee pain, which is relieved with acetaminophen. On physical examination, he walks with a mild limp on the right. He has no swelling or tenderness over the right lower extremity. Range of motion of the right knee is full and painless. Range of motion testing of the right hip reveals guarding and limited internal rotation. Which of the following is the most likely diagnosis? ALegg-Calvé-Perthes disease BSlipped capital femoral epiphysis CStress fracture of the femoral neck DTransient synovitis of the hip

Legg-Calvé-Perthes disease The most likely diagnosis is Legg-Calvé-Perthes disease or avascular necrosis of the femoral head in a growing child of unknown etiology. It is most common in children between the ages of 5 and 8 and is more common in boys. The pathophysiology involves disruption of the blood supply to the femoral head, which results in necrosis of bone, marrow, and cartilage. The femoral epiphysis becomes flattened, fragmented, and more dense due to compression of bony trabeculae and resorption of the necrotic bone. Spontaneous revascularization eventually occurs, but deformity of the femoral head may persist. Children present with a limpand activity-related pain of gradual onset and often complain of knee pain. It is important to examine the hip in any child with knee pain. Physical examination findings include a limp and limited range of motion of the hip, especially internal rotation. Patients with advanced disease may have severely restricted hip motion, muscle atrophy, or a leg length discrepancy. X-rays may be normal early in the disease, then later show increased density and smaller size of the epiphysis, followed by fragmentation of the epiphysis, and eventual revascularization and healing. Technetium bone scanning can be used to diagnose early Legg-Calvé-Perthes disease before X-ray changes appear, but magnetic resonance imaging has mostly replaced it as a diagnostic test. Treatment involves maintaining hip range of motion and containing the femoral head in the acetabulumthrough physical therapy, bracing, or surgery. The prognosis correlates with the age at diagnosis. Children younger than 6 years old have a better prognosis because they have more growth remaining and more time for the femoral head to remodel than older children who are more likely to require surgery. Over the long term, the femoral head deformity results in an incongruent joint, and over half of patients will develop significant osteoarthritis by age 60.

A 7-year-old boy presents to the office with his mother, who states that he has been limping on and off for the past three months. Occasionally, he complains of right knee pain, which is relieved with acetaminophen. On physical examination, he walks with a mild limp on the right. He has no swelling or tenderness over the right lower extremity. Range of motion of the right knee is full and painless. Range of motion testing of the right hip reveals guarding and limited internal rotation. Which of the following is the most likely diagnosis? ALegg-Calvé-Perthes disease BSlipped capital femoral epiphysis CStress fracture of the femoral neck DTransient synovitis of the hip

Legg-Calvé-Perthes disease The most likely diagnosis is Legg-Calvé-Perthes disease or avascular necrosis of the femoral head in a growing child of unknown etiology. It is most common in children between the ages of 5 and 8 and is more common in boys. The pathophysiology involves disruption of the blood supply to the femoral head, which results in necrosis of bone, marrow, and cartilage. The femoral epiphysis becomes flattened, fragmented, and more dense due to compression of bony trabeculae and resorption of the necrotic bone. Spontaneous revascularization eventually occurs, but deformity of the femoral head may persist. Children present with a limpand activity-related pain of gradual onset and often complain of knee pain. It is important to examine the hip in any child with knee pain. Physical examination findings include a limp and limited range of motion of the hip, especially internal rotation. Patients with advanced disease may have severely restricted hip motion, muscle atrophy, or a leg length discrepancy. X-rays may be normal early in the disease, then later show increased density and smaller size of the epiphysis, followed by fragmentation of the epiphysis, and eventual revascularization and healing. Technetium bone scanning can be used to diagnose early Legg-Calvé-Perthes disease before X-ray changes appear, but magnetic resonance imaging has mostly replaced it as a diagnostic test. Treatment involves maintaining hip range of motion and containing the femoral head in the acetabulumthrough physical therapy, bracing, or surgery. The prognosis correlates with the age at diagnosis. Children younger than 6 years old have a better prognosis because they have more growth remaining and more time for the femoral head to remodel than older children who are more likely to require surgery. Over the long term, the femoral head deformity results in an incongruent joint, and over half of patients will develop significant osteoarthritis by age 60.

Which of the following physical examination findings is most suggestive of adhesive capsulitis of the shoulder? ACrepitus with range of motion BLimited active and passive range of motion CLimited active more than passive range of motion DWeakness with resisted external rotation with the arm at the side

Limited active and passive range of motion

A 25-year-old woman presents to the emergency department after she injured her right foot playing soccer. X-rays show a fracture of the fifth metatarsal. Which of the following is the most important piece of information to help you formulate a treatment plan? AFracture comminution BFracture displacement CLocation of the fracture in the bone DOverlap of fracture ends

Location of the fracture in the bone

A 63-year-old man with a history of atrial fibrillation is evaluated at his bedside in the intensive care unit. He is unable to move his extremities, speak, or swallow. Blinking and vertical eye movements are preserved. Which of the following is the most likely diagnosis? AAkinetic mutism BComa CLocked-in syndrome DVegetative state

Locked-in syndrome Locked-in syndrome is a state of quadriplegia and paralysis of oral structures resulting in the loss of speechand swallow. Patients retain their alertness and cognitive functioning, as well as their ability to blink and move their eyes vertically due to the sparing of the lateral and medial tegmentum. Stroke, either ischemic or hemorrhagic, is the most common cause of locked-in syndrome and typically occurs due to embolism within the basilar artery. Other causes of locked-in syndrome include trauma, pontine abscess, brainstem tumors, central pontine myelinolysis, and heroin abuse. Patients may exhibit various involuntary movements, which can include crying, trismus, grimacing, palatal myoclonus, bruxism, laughing, shivering, and dystonic posturing. Physical examination should focus on vertical eye movements so that patients are not misdiagnosed as having a comatose state. Neuroimaging, particularly magnetic resonance imaging, is necessary to detect evidence of infarction or lesion to the pons. Magnetic resonance angiography is also beneficial to identify vascular occlusion or severe stenosis within intracranial arteries. In the event of a persistent vascular occlusion, treatment is aimed at thrombolysis and recanalization of the affected vessel. Patients may regain some function over time. Aggressive physical rehabilitation and supportive therapy are necessary.

Which of the following is the most common site for development of an Ewing sarcoma tumor? AIrregular bones BLong bones CSesamoid bones DShort bones

Long bones

A 19-year-old man presents to the emergency department after being hit in the temporal region of the head with a baseball bat. CT scan of the head without contrast shows a lens-shaped hemorrhage. He is diagnosed with an epidural hematoma. Which of the following is a classic clinical finding of epidural hematomas? AChronic headaches BLoss of consciousness followed by a lucid interval CResting tremor DStocking-glove paresthesias

Loss of consciousness followed by a lucid interval Epidural hematomas arise in the potential space between the dura mater and the skull and most commonly occur in adolescents and young adults. The most common cause is head trauma, which frequently occurs due to a motor vehicle collision, fall, or assault. Atraumatic epidural hematoma is rare. The source of blood in epidural hematomas is usually arterial, but 15% of cases are due to venous bleeding. The major cause of arterial injury is trauma to the sphenoid bone causing injury to the middle meningeal artery. Patients with epidural hematoma present with a history of head trauma and a variety of clinical manifestations. Historical features that increase the risk of epidural hematoma include high risk trauma mechanisms (fall from a height greater than 10 feet), mechanisms involving a direct blow to the temporal or occipital region, symptoms of increased intracranial pressure, head trauma in patients with a bleeding disorder or anticoagulation use, and ahistory of sickle cell disease. In children, direct blows to the temporal or occipital region should raise concern even in well-appearing children. Even minor mechanisms of head injury in young infants can cause serious traumatic brain injury. Symptoms of increased intracranial pressure include headache, vomiting, seizure, confusion, lethargy, or loss of consciousness and are associated with epidural hematoma and other intracranial injuries. The classic presentation of epidural hematomas is a loss of consciousness followed by a lucid interval for a period that ranges from minutes to hours (typically four to six hours) prior to sudden onset of clinical deterioration and sometimes coma. Clinical deterioration occurs in these cases due to continued arterial bleeding and subsequent hematoma expansion. Some patients have focal neurologic deficits, such as aphasia or hemiparesis. Skull fractures are present in 80% of patients with an epidural hematoma. Exam findings of increased intracranial pressure may include ipsilateral dilated pupil caused by uncal (innermost part of the temporal lobe) herniation with compression of the oculomotor nerve. The initial imaging study used in patients with head trauma is a CT of the head without contrast. The characteristic finding on CT is a lens-shaped (biconvex) hemorrhage because the epidural hematoma is limited by firm dural attachments at the cranial sinuses. Neuroimaging is crucial for both diagnosing and guiding initial management. Acute head trauma is associated with other intracranial pathology, such as subdural hematoma, subarachnoid hemorrhage, cerebral contusion, and diffuse brain swelling. Lumbar puncture is contraindicated in cases with a space-occupying lesion, such as an epidural hematoma, due to the risk of herniation and death. The management of an epidural hematoma depends on the patient's neurologic status, pupillary findings, hematoma volume, and the degree of midline shift. The most common management is surgical management with hematoma evacuation. Craniotomy and hematoma evacuation is the mainstay of surgical treatment of symptomatic acute epidural hematoma. Surgery is performed within one or two hours of presentation in patients with neurologic deterioration and an epidural hematoma. Patients with a small hematoma and mild symptoms can be managed nonoperatively by monitoring for neurologic deterioration and frequently repeating brain imaging to check for hematoma expansion. Glucocorticoid therapy is not indicated following head injury and may be associated with increased acute mortality. The mortality rate for epidural hematomas is about 5-10%. Factors that help predict the prognosis include the severity of the neurologic deficit (determined by the Glasgow coma scale), pupillary abnormalities, hematoma volume, and the degree of midline shift.

A 27-year-old woman with complaints of fatigue and paresthesias has magnetic resonance imaging of her brain with gadolinium showing evidence of ovoid-shaped demyelinating lesions in periventricular areas of her brain. To confirm the diagnosis of multiple sclerosis, which of the following diagnostic studies should be considered? AEvoked potentials BLumbar puncture CNerve conduction study DSerum oligoclonal band leve

Lumbar puncture Evoked potentials (A) is incorrect. Evoked potentials can identify subclinical lesions in multiple sclerosis. Nerve conduction study (C) is incorrect. While helpful, nerve conduction study is not required for diagnosis of multiple sclerosis. Serum oligoclonal band level (D) is incorrect. Oligoclonal bands are evaluated in cerebrospinal fluid but are not performed with routine phlebotomy. Lumbar puncture is utilized in the workup of multiple sclerosis to identify evidence of oligoclonal bands and immunoglobulin G. Multiple sclerosis is an inflammatory demyelinating disease of the nervous system. Multiple sclerosis can be relapsing-remitting, primary progressive, and secondary progressive. The disease is more common in women and typically occurs between the ages of 20-40 years old. Symptoms vary depending on the area of the brain affected. More common presenting complaints include fatigue, weakness, paresthesias, and visual disturbances. Patients can also develop bladder and sexual dysfunction, incoordination, and impaired ability to ambulate. Patients presenting with optic neuritis, transverse myelitis, or internuclear ophthalmoplegia should be considered for complete multiple sclerosis workup given the association with the disorder. Physical examination often demonstrates brisk deep tendon reflexes, dysdiadochokinesia, positive Babinski sign, dysmetria, and nystagmus. Magnetic resonance imaging of the brain with gadolinium contrast should be ordered in all patients in whom multiple sclerosis is suspected. Ovoid-shaped plaquessurrounding the corpus callosum and periventricular areas of the brain can be seen. Follow-up cervical spine and thoracic spine imaging is often ordered as demyelinating plaques can also occur at these levels in the spinal cord. Cerebrospinal fluid analysis demonstrates oligoclonal bands, elevated protein, and increased immunoglobulin G. Disease-modifying therapies are used to reduce risk of relapses. Symptomatic treatment is used in conjunction with disease-modifying therapies to address underlying symptoms of fatigue, incontinence, erectile dysfunction, and spasticity. Patients presenting with an acute exacerbation should receive intravenous corticosteroids at a rate of 1 g per day for five consecutive days.

A 25-year-old man is brought to the emergency department with syncope following sudden onset of the "worst headache of my life." CT scan of the head is ordered, which does not demonstrate evidence of intraventricular blood. What would be the next step to effectively rule out subarachnoid hemorrhage? ACerebral angiography BLumbar puncture CMagnetic resonance imaging of the brain DRepeat CT scan in 24 hours

Lumbar puncture Lumbar puncture is required in the evaluation of a patient with presumptive subarachnoid hemorrhage. Subarachnoid hemorrhage is bleeding within the subarachnoid space often a result of a ruptured saccular aneurysm. Risk factors include cigarette smoking and hypertension. Patients commonly present complaining of sudden onset of the "worst headache of my life." The hemorrhage can be precipitated by physical exertion, such as with bowel movements, exercise, and sexual intercourse. Patients may have associated syncope, vomiting, and signs of meningeal irritation. Patients should undergo a noncontrast CT scan of the brain to evaluate for evidence of intraventricular hemorrhage. If a CT scan is negative for subarachnoid hemorrhage, a lumbar puncture needs to be performed to effectively rule out the diagnosis. Hemoglobin degradation products (xanthochromia) is highly suggestive of subarachnoid hemorrhage. Elevated opening pressure and elevated red blood cell counts that do not diminish with each collection tube are supportive of the diagnosis. Angiography can be performed after diagnosis is confirmed to evaluate for evidence of ruptured or leaking aneurysm. Patients with evidence of aneurysm may need to undergo aneurysmal coiling or clipping. Complications include vasospasm causing delayed cerebral ischemia, rebleeding, seizures, and death.

A 55-year-old man presents with new-onset seizures and frontal headaches. Past medical history is significant for neurofibromatosis. Physical examination reveals papilledema and motor weakness. Magnetic resonance imaging shows a calcified tumor in the white matter of the frontal lobe. Biopsy of the tumor reveals a "fried-egg" appearance on histology. Which of the following is the most likely diagnosis? AEpendymoma BGlioblastoma multiforme CMedulloblastoma DOligodendroglioma

Oligodendroglioma Brain tumors are the benign or malignant proliferation of central nervous system tissues. Examples of benign brain tumors include meningioma, schwannoma, and pilocytic astrocytoma. Examples of malignant brain tumors include glioblastoma multiforme, oligodendroglioma, medulloblastoma, and ependymoma. Brain tumors may also represent metastatic lesions from a distant site including the lungs, breast, skin (melanoma), and colon. Tumors arising directly from the brain tissue are called primary brain tumors and are classified according to the cell type of origin, including astrocytes, meningothelial cells, ependymal cells, oligodendrocytes, and neuroectoderm. In adults, the majority of primary brain tumors are supratentorial, while in children, the majority of brain tumors are usually infratentorial, with the most common tumors in children being pilocytic astrocytoma, ependymoma, and medulloblastoma. Tumors that originate extracranially are called metastatic tumors and present as multiple, well-circumscribed lesions seen at the junction of gray-white matter. Oligodendroglioma is a malignant tumor of the oligodendrocytes that are present in the frontal lobe. Brain tumors, in general, may cause invasion of the brain tissues, edema, and hydrocephalus that elevate the intracranial pressure and impair cerebral perfusion. Risk factors for brain tumors include hereditary diseases, such as neurofibromatosis, multiple endocrine neoplasia type 1, and tuberous sclerosis; human immunodeficiency virus; and neck and head irradiation. Patients may present with headaches, altered mentation, ataxia, weakness, focal seizures, fixed visual changes, speech deficits, and focal sensory abnormalities. Patients with oligodendroglioma usually present with seizures, motor weakness, and cognitive decline. Physical examination may reveal focal or generalized deficit, papilledema, diplopia, and visual field defects. Anosmia, cranial nerve palsies, or nystagmus may also be present. Laboratory studies for evaluating brain tumors may include complete blood count, coagulation studies, electrolyte studies, and comprehensive metabolic panel. Magnetic resonance imaging is the imaging modality of choice and may show a calcified tumor in the white matter of the frontal lobe in a patient presenting with oligodendroglioma. Computed tomography and positron emission tomography may also aid in diagnosing brain tumors. Rarely, brain tumors may be biopsied. A biopsy may show a "fried-egg" appearance in a patient with oligodendroglioma. Treatment is multidisciplinary involving surgical debulking, ventricular shunt installation, and radiation therapy.

Which of the following diagnostic studies is most notably abnormal in a patient with pseudotumor cerebri? ACT of brain BElectroencephalogram CLumbar puncture DMRI of brain

Lumbar puncture Pseudotumor cerebri refers to idiopathic elevated intracranial pressure. Obese women of childbearing years are most commonly affected. Presenting signs and symptoms of pseudotumor cerebri include headache, photopsia, visual disturbances or visual loss (transient or permanent), back pain, pain behind the eyes, and intracranial noises. On physical exam, these patients may have visual field deficits, sixth cranial nerve(abducens) palsy, or papilledema. The diagnosis of pseudotumor cerebri is one of exclusion. Lumbar punctureshould be performed on all patients with suspected increased intracranial pressure. The lumbar puncture of a patient with pseudotumor cerebri will show elevated opening pressure with normal cerebrospinal fluid analysis. MRI or CT scan of the brain should be ordered and must show no increase in ventricle size and no masses or other parenchymal changes. Ophthalmologic exam may reveal papilledema. For diagnosis of pseudotumor cerebri, the patient must have no other systemic causes of increased intracranial pressure, no neurologic deficits besides cranial nerve palsies, and no decreased level of consciousness. Treatment of pseudotumor cerebri involves losing weight, discontinuing any offending drugs (such as tetracyclines), and administering carbonic anhydrase inhibitors. Acetazolamide and others of its class inhibit carbonic anhydrase, which in turn decreases production of cerebrospinal fluid. Furosemide is also sometimes used along with acetazolamide to decrease cerebrospinal fluid levels. Patients who are intolerant to medical treatment can undergo serial lumbar punctures or surgical procedures such as shunting.

A 55-year-old woman presents to the clinic with a 10-month history of extremely dry eyes that prevent her from wearing her contacts. In addition, she notes that four months ago she began noticing a "cotton mouth" sensation and difficulty swallowing foods. Which of the following most accurately describes the pathophysiology of the suspected diagnosis? ALymphocytic infiltrate in the endocrine glands BLymphocytic infiltrate in the exocrine glands CNeutrophilic infiltrate in the endocrine glands DNeutrophilic infiltrate in the exocrine glands

Lymphocytic infiltrate in the exocrine glands Lymphocytic infiltrate in the exocrine glands is characteristic of the pathophysiology of Sjögren syndrome. Sjögren syndrome is a systemic autoimmune disorder that causes dysfunction of the lacrimal and salivary glands. The condition is nine times more common in women and occurs most commonly between 40 and 60 years of age. Sjögren syndrome can occur in isolation, termed primary Sjögren syndrome, or in association with other rheumatologic diseases, termed secondary Sjögren syndrome. Patients with Sjögren syndrome present with keratoconjunctivitis sicca resulting from inadequate tear production caused by lymphocyte and plasma cell infiltrate of the lacrimal glands.

Which of the following diagnostic studies is most effective for evaluating tumor size in a patient with chondrosarcoma? ABone density scan BMagnetic resonance imaging CPlain radiograph DUltrasound

Magnetic resonance imaging Chondrosarcoma is a term used to describe a group of bone tumors that are made up of a chondroid or cartilaginous matrix. The tumors can range from low-grade, slow-growing tumors with low metastatic potential to high-grade tumors that are aggressive and have a high metastatic potential. If there are any bone-forming elements or primitive mesenchymal elements found in the tumor, then it should not be classified as a chondrosarcoma, which is made up of cartilage alone. Conventional central chondrosarcomas make up the majority of these tumors and affect the axial skeleton with the pelvis and ribs being the most common site. The age of peak incidence for conventional central chondrosarcomas is 50-70 years. Other types of chondrosarcomas have varying ages of incidence. There is no difference in incidence based on ethnic group. The male to female ratio depends on the type of tumor involved, with most being equal. The most common presenting symptom of chondrosarcoma is a deep, dull pain that occurs at night. Swelling may also be present at the site of the tumor. If the tumor is close to a neurovascular bundle, such as in the pelvis, neuropathy may occur. If it is close to a joint, range of motion and function may be affected. Patients may also present due to pathologic fracture of the affected bone. Workup for chondrosarcoma typically relies on the use of diagnostic imaging. Magnetic resonance imaging (MRI) or computed tomography is used to evaluate tumor size and local extent of the tumor. MRI can also help determine the extent of bone marrow and soft tissue involvement in the disease. MRI is also used to diagnose or confirm the return of chondrosarcoma at a surgically-treated site. Treatment involves surgery for all types of chondrosarcomas. Chemotherapy and radiation therapy play limited roles in the primary treatment of the disease,but may be used in select cases and for palliative care. Prognosis depends on the grade and stage of the tumor at initial diagnosis.

Which of the following diagnostic studies is most effective for evaluating tumor size in a patient with chondrosarcoma? ABone density scan BMagnetic resonance imaging CPlain radiographYour Answer DUltrasound

Magnetic resonance imaging Chondrosarcoma is a term used to describe a group of bone tumors that are made up of a chondroid or cartilaginous matrix. The tumors can range from low-grade, slow-growing tumors with low metastatic potential to high-grade tumors that are aggressive and have a high metastatic potential. If there are any bone-forming elements or primitive mesenchymal elements found in the tumor, then it should not be classified as a chondrosarcoma, which is made up of cartilage alone. Conventional central chondrosarcomas make up the majority of these tumors and affect the axial skeleton with the pelvis and ribs being the most common site. The age of peak incidence for conventional central chondrosarcomas is 50-70 years. Other types of chondrosarcomas have varying ages of incidence. There is no difference in incidence based on ethnic group. The male to female ratio depends on the type of tumor involved, with most being equal. The most common presenting symptom of chondrosarcoma is a deep, dull pain that occurs at night. Swelling may also be present at the site of the tumor. If the tumor is close to a neurovascular bundle, such as in the pelvis, neuropathy may occur. If it is close to a joint, range of motion and function may be affected. Patients may also present due to pathologic fracture of the affected bone. Workup for chondrosarcoma typically relies on the use of diagnostic imaging. Magnetic resonance imaging (MRI) or computed tomography is used to evaluate tumor size and local extent of the tumor. MRI can also help determine the extent of bone marrow and soft tissue involvement in the disease. MRI is also used to diagnose or confirm the return of chondrosarcoma at a surgically-treated site. Treatment involves surgery for all types of chondrosarcomas. Chemotherapy and radiation therapy play limited roles in the primary treatment of the disease,but may be used in select cases and for palliative care. Prognosis depends on the grade and stage of the tumor at initial diagnosis.

A 65-year-old woman presents to the emergency department after falling in her home. She complains of pain with breathing, especially inspiration. Physical exam reveals tenderness to palpation of the chest wall. The patient is not in distress. A chest X-ray is ordered and shows a single rib fracture. Which of the following is the most appropriate treatment? AIntercostal nerve block BOral analgesics CRib binder DSurgical fixation

Oral analgesics

A 62-year-old woman presents to the office with scalp tenderness on the left side of her head, double vision, and pain with chewing that started about two days ago and has gotten worse. Physical exam reveals a tender and enlarged left temporal artery, and you suspect giant cell arteritis. Which of the following is the most appropriate treatment for this condition? AAspirin BMethotrexate CNonsteroidal anti-inflammatories DPrednisone

Prednisone Aspirin (A) has been shown to be helpful as an addition to prednisone to reduce the chance of visual loss and stroke but should not replace prednisone as the first-line treatment. Methotrexate (B) has not been proven to be as effective as prednisone. Nonsteroidal anti-inflammatories (C) are not a treatment for giant cell arteritis.

A 28-year-old woman experiences chronic migraine headaches that are not fully controlled with abortive medications. She prefers to avoid prophylactic medications and would like other suggestions to help prevent these episodes. Which of the following lifestyle modifications is most helpful in preventing further migraines? ABegin oral contraceptives BIncrease daily caffeine intake CIncrease frequency of abortive medications DMaintain proper sleep and diet

Maintain proper sleep and diet Begin oral contraceptives (A) is incorrect, since oral contraceptives have been shown to trigger migraine headaches. More importantly, oral contraceptive pills are contraindicated in patients who experience migraine headaches with aura due to the increased risk of cardiovascular conditions and ischemic stroke. Oral contraceptive pills must also be discontinued in patients with migraines who begin to experience aura symptoms. For patients over 35 years of age who experience migraine headaches without aura, cardiovascular risk factors should be carefully considered before prescribing oral contraceptives. Increase daily caffeine intake (B) is incorrect. Although some abortive medications may contain caffeine, increasing daily caffeine intake is not recommended and may even trigger headaches. Increase frequency of abortive medications (C) is incorrect as this can lead to medication overuse when analgesics are used more than 15 days per month and combined analgesics are used more than 10 days per month. Overuse of these medications will cause rebound headaches and an overall worsening of the condition. Patients who are not controlled on abortive medications less than 15 days per month require prophylactic control with long-term medications.

Which of the following is the most likely diagnosis seen in the above radiographs? AGaleazzi fracture BMaisonneuve fracture CPilon fracture DTibial plateau fracture

Maisonneuve fracture A Maisonneuve fracture is defined as a spiral fracture of the proximal third of the fibula in association with a distal medial malleolar fracture or deltoid ligament rupture. It results from a pronation and external rotation injury of the ankle which disrupts the tibiofibular syndesmosis and interosseous membrane, transmitting an upward and lateral force to the proximal fibula. Patients present with medial ankle pain and instability as well as pain over the proximal aspect of the fibula. Diagnosis is made with plain radiographs of both the distal ankle and proximal fibula. Due to the risk of this injury being missed in patients who present primarily with ankle pain, there should be a high clinical suspicion to ensure that further imaging of the proximal fibula is performed. On X-ray, medial ankle injury is depicted as either a distal medial malleolar fracture or by widening of the ankle joint. Treatment is with surgical fixation of the distal ankle joint with syndesmotic screws.

A patient presents with paresthesia in the distribution of the first three digits and radial half of the fourth digit of the hand. Symptoms worsen with flexion or extension of the wrist. Impairment of which of the following is most likely? AAxillary nerve BMedian nerve CMusculocutaneous nerve DRadial nerve EUlnar nerve

Median nerve The patient has carpal tunnel syndrome, which is caused by compression of the median nerve as it passes through the carpal tunnel. This tunnel is formed by the bones of the wrist and the carpal ligament (flexor retinaculum). Carpal tunnel syndrome is a common disorder and usually presents with paresthesia in the first three fingers and radial half of the fourth finger, along with pain in the wrist and palm. Symptoms are worsened with flexion or extension of the wrist, which puts further pressure on the compressed median nerve. Certain maneuvers during examination can help to confirm the diagnosis. The Phalen test asks the patient to flex the wrist with the elbow extended or to place the backs of the hands together in order to cause hyperflexion of the wrist. Pain in the fingers innervated by the median nerve is considered to be a positive Phalen sign. The clinician can also percuss the wrist over the distribution of the carpal tunnel; if pain or paresthesia occurs in the fingers innervated by the median nerve, this is considered to be a positive Tinel test. Nerve conduction studies and electromyography are also highly specific and sensitive in confirming the diagnosis. Treatment can involve wrist splinting, steroid injections, or, if these more conservative measures fail, surgery for carpal tunnel release.

A 35-year-old man presents to the emergency department following the sudden onset of a severe headache. Which of the following characteristics is more likely to develop several hours after the initial bleed in a subarachnoid hemorrhage? AHeadache BLoss of consciousness CMeningismus DVomiting

Meningismus Meningismus may develop several hours after the initial bleeding in a patient with subarachnoid hemorrhage. This characteristic results from the inflammation of the meninges that occurs as blood products are broken down in the cerebrospinal fluid. Subarachnoid hemorrhage is an uncommon cause of stroke with a high mortality rate. It most often occurs as a result of a ruptured saccular aneurysm but can also occur secondary to trauma, other cerebrovascular abnormalities, or cocaine use. It typically occurs between 40 and 60 years of age. Risk factors include high blood pressure, cigarette smoking, polycystic kidney disease, Ehler-Danlos syndrome, and cocaine and methamphetamine usage. When a saccular aneurysm ruptures, the blood it releases rapidly increases intracranial pressure due to arterial bleeding within the cerebrospinal fluid. The most common presentation of a subarachnoid hemorrhage is a sudden onset of a thunderclap headache, often associated with loss of consciousness, vomiting, and neck stiffness. Physical examination can reveal elevated blood pressure, signs suggestive of meningeal irritation, and preretinal hemorrhages. Depending on the location of rupture and bleeding, more focal neurological findings, such as third nerve palsy, can also be demonstrated. The severity of a subarachnoid hemorrhage is graded based on the Hunt and Hess grading system. The initial diagnostic workup must include a CT scan of the head without contrast to evaluate for presence of bleeding. If the CT scan is negative, then a lumbar puncture must be performed to definitively rule out a subarachnoid hemorrhage. Classic lumbar puncture findings include persistently elevated red blood cells throughout all tubes, xanthochromia, and an elevated opening pressure. Once the diagnosis is confirmed with initial testing, angiography (CT angiography, magnetic resonance angiography, or conventional angiography) should be performed to detect aneurysms and determine best intervention. Patients should be emergently stabilized and may need endotracheal intubation. Prevention of rebleeding is a mainstay of treatment, and aneurysms should be clipped or coiled depending on their location. Seizures should be treated with levetiracetam, carbamazepine, or phenobarbital. Complications include hydrocephalus, vasospasm, and delayed cerebral ischemia. Vasospasm is treated with calcium channel blockers, particularly nimodipine. For patients with hydrocephalus, an external ventricular drain is placed to drain cerebrospinal fluid and reduce intracranial pressure.

A 35-year-old woman presents to the office with pain in her left foot that has been gradually worsening over the past three weeks. She started training for a half marathon six weeks ago and had been running up to 25 miles per week. When the pain started, she was able to "run through" it, but now she notes pain that steadily worsens as she continues to run, and she has to stop. On physical examination, she is tender over the dorsum of the foot in the midfoot area. She has pain with axial loading of the third toe. X-rays of the foot are normal. Which of the following is the most likely diagnosis? AMetatarsal stress fracture BMetatarsalgia CMorton neuroma DPlantar fasciitis

Metatarsal stress fracture Metatarsal stress fractures are overuse injuries that occur from repetitive stress to the forefoot. They were originally described in military recruits ("march fractures"). They are often seen in runners and may be related to an increase in duration or intensity or improper footwear. Other risk factors for metatarsal stress fractures include variations in foot anatomy, tight gastrocnemius muscles, pes planus, obesity, osteopenia, and muscle weakness. Patients present with the gradual onset of pain in the foot that is worse with use. Initially, the pain may be mild and intermittent, but with continued use, it progresses to the point where it may interfere with activities or result in a complete fracture. Physical examination findings include localized tenderness over the metatarsal shaft and pain with axial loading of the corresponding toe (the toe is pushed proximally along the axis of the metatarsal shaft). Swelling may be present but is usually mild unless the stress fracture is advanced. X-rays are often normal, as changes do not appear for two to six weeks. The diagnosis is primarily clinical, and advanced imaging is not necessary, but magnetic resonance imaging or technetium bone scanning may be used to confirm the diagnosis. Ultrasound has also been shown to be accurate in diagnosing stress fractures. Treatment is avoiding the aggravating activity and implementing symptomatic measures such as ice and acetaminophen. Immobilization is not necessary, but if patients have pain with walking, they can use a postoperative shoe or a shoe with a stiff sole. Patients may participate in low-impact activities as tolerated until pain free, about four to eight weeks, and then may gradually resume full activities. An exception is a stress fracture of the proximal fifth metatarsal. These fractures are rare, but the prognosis is similar to a Jones fracture, with a higher incidence of nonunion. These fractures are treated with immobilization and nonweight bearing or surgical fixation.

Which of the following medications used to treat rheumatoid arthritis has been shown to alter the disease course and improve radiographic outcomes? AIbuprofen BMethotrexate CNaproxen DPrednisone

Methotrexate Methotrexate, a disease-modifying antirheumatic drug, is considered the first-line drug of this class used to treat rheumatoid arthritis. Other drugs in this class include hydroxychloroquine, sulfasalazine, leflunomide, and the tumor necrosis factor inhibitors, among others. Rheumatoid arthritis is a chronic systemic inflammatory disease of unknown etiology that causes a symmetric polyarthritis that most commonly affects the small joints of the hands and feet. It most commonly affects the proximal interphalangeal joints and the metacarpophalangeal joints, while sparing the distal interphalangeal joints. In addition, the wrists, knees, ankles and metatarsophalangeal joints may also be affected. The arthritis typically leads to destruction of joints due to erosion of cartilage and bone. Patients experience an insidious onset of joint pain and swelling that interferes with daily function, with morning stiffness lasting more than 30 minutes. Patients may also demonstrate extra-articular manifestations of disease including subcutaneous rheumatoid nodules, interstitial lung disease, pleural effusion, pericarditis, splenomegaly, and blood abnormalities. Physical exam reveals swollen, tender, erythematous "boggy" joints and rheumatoid nodules. Commonly, ulnar deviation is seen at the metacarpophalangeal joints. Flexion of the proximal interphalangeal joint and hyperextension of the distal interphalangeal joint leads to Boutonniere deformity, while flexion of the distal interphalangeal joint and hyperextension at the proximal interphalangeal joint leads to swan neck deformity. Diagnosis is based on radiographic and laboratory findings. Radiographs demonstrate joint erosions, joint space narrowing, and osteoporotic changes. Laboratory tests reveal positive rheumatoid factor and antibodies to cyclic citrullinated peptides (anti-CCP) in 70-80% of patients affected. Treatment includes a variety of pharmacologic options. While the nonsteroidal anti-inflammatory drugs and disease-modifying antirheumatic drugs have been shown to improve the symptoms of rheumatoid arthritis, only the disease-modifying antirheumatic drug class has been shown to alter the disease course and improve radiographic outcomes. Methotrexate is generally well tolerated in these patients and produces beneficial effects in two to six weeks. Methotrexate should be taken with folic acid supplementation due to its mechanism of inhibiting dihydrofolate reductase and serving as a folate antagonist. During the first few weeks that the patient is started on methotrexate, nonsteroidal anti-inflammatory drugs and corticosteroids may be given to control symptoms and reduce disease activity until the methotrexate takes effect.

Which of the following medications used to treat rheumatoid arthritis has been shown to alter the disease course and improve radiographic outcomes? AIbuprofen BMethotrexate CNaproxen DPrednisone

Methotrexate Methotrexate, a disease-modifying antirheumatic drug, is considered the first-line drug of this class used to treat rheumatoid arthritis. Other drugs in this class include hydroxychloroquine, sulfasalazine, leflunomide, and the tumor necrosis factor inhibitors, among others. Rheumatoid arthritis is a chronic systemic inflammatory disease of unknown etiology that causes a symmetric polyarthritis that most commonly affects the small joints of the hands and feet. It most commonly affects the proximal interphalangeal joints and the metacarpophalangeal joints, while sparing the distal interphalangeal joints. In addition, the wrists, knees, ankles and metatarsophalangeal joints may also be affected. The arthritis typically leads to destruction of joints due to erosion of cartilage and bone. Patients experience an insidious onset of joint pain and swelling that interferes with daily function, with morning stiffness lasting more than 30 minutes. Patients may also demonstrate extra-articular manifestations of disease including subcutaneous rheumatoid nodules, interstitial lung disease, pleural effusion, pericarditis, splenomegaly, and blood abnormalities. Physical exam reveals swollen, tender, erythematous "boggy" joints and rheumatoid nodules. Commonly, ulnar deviation is seen at the metacarpophalangeal joints. Flexion of the proximal interphalangeal joint and hyperextension of the distal interphalangeal joint leads to Boutonniere deformity, while flexion of the distal interphalangeal joint and hyperextension at the proximal interphalangeal joint leads to swan neck deformity. Diagnosis is based on radiographic and laboratory findings. Radiographs demonstrate joint erosions, joint space narrowing, and osteoporotic changes. Laboratory tests reveal positive rheumatoid factor and antibodies to cyclic citrullinated peptides (anti-CCP) in 70-80% of patients affected. Treatment includes a variety of pharmacologic options. While the nonsteroidal anti-inflammatory drugs and disease-modifying antirheumatic drugs have been shown to improve the symptoms of rheumatoid arthritis, only the disease-modifying antirheumatic drug class has been shown to alter the disease course and improve radiographic outcomes. Methotrexate is generally well tolerated in these patients and produces beneficial effects in two to six weeks. Methotrexate should be taken with folic acid supplementation due to its mechanism of inhibiting dihydrofolate reductase and serving as a folate antagonist. During the first few weeks that the patient is started on methotrexate, nonsteroidal anti-inflammatory drugs and corticosteroids may be given to control symptoms and reduce disease activity until the methotrexate takes effect.

Which of the following best describes the primary anatomic change in amyotrophic lateral sclerosis? AMotor neuron degeneration BOccipital lobe neuronal atrophy CParietal lobe neuronal atrophy DSensory neuron degeneration

Motor neuron degeneration Amyotrophic lateral sclerosis is a progressive neurologic disorder primarily involving the degeneration of motor neurons. The cause is sporadic and unknown in most cases, although a hereditary form exists, and some research suggests exposure to certain environmental toxins may play a role. Symptoms usually begin in mid-to-late adulthood and increase in prevalence and severity with age. Men are slightly more likely to be affected by amyotrophic lateral sclerosis, as are those of Caucasian race. Presenting symptoms are varied but most commonly involve muscle weakness as well as spasticity. Patients will manifest symptoms of both lower motor neuron and upper motor neuron malfunction. Lower motor neuron symptoms include limb weakness, dysarthria, dysphagia, poor posture, and dyspnea. Upper motor neuron involvement manifests as stiffness, incoordination, poor gait, poor balance, and muscle spasms. Frontotemporal dementia and atrophy can also occur, leading to personality changes and language difficulties. The evolution of amyotrophic lateral sclerosis is progressive and linear without episodes of remission and relapse. Median survival after diagnosis is three to five years. Diagnosis of amyotrophic lateral sclerosis is made clinically and is based on the presence of both upper and lower motor neuron dysfunction, a progressive decline in function without intervals of remission, and absence of any other neurologic explanation for the patient's symptoms. There is no curative treatment, but one drug exists that has been shown to prolong survival, which is riluzole. Riluzole is thought to benefit those with amyotrophic lateral sclerosis through inhibition of glutamate and through direct action on sodium channels.

A 31-year-old woman presents to the office with concerns regarding her left eye and jaw. Over the past three months, she has noted ptosis of her left eye developing later in the day and is now experiencing associated diplopia and fatigable chewing. Which of the following diagnoses is of primary concern? AAmyotrophic lateral sclerosis BMultiple sclerosis CMyasthenia gravis DOculopharyngeal muscular dystrophy

Myasthenia gravis

Which of the following pharmacologic therapies is generally a first-line treatment for osteoarthritis? AAcetaminophen BHyaluronic acid CHydrocodone DNaproxen

Naproxen Acetaminophen (A) has a negligible or non-clinically significant impact on osteoarthritis pain compared to placebo. It is therefore no longer a first-line therapy. Hyaluronic acid (B) has had mixed results in studies. It is given as an injection in some cases of knee osteoarthritis but has shown minimal clinical benefit compared to placebo. Hydrocodone (C) is an opioid. Due to the high incidence of side effects and the abuse potential, opioids are generally not recommended for the treatment of osteoarthritis. One possible exception is in patients with severe pain awaiting joint replacement.

Which of the following is true regarding the treatment of an aneurysmal subarachnoid hemorrhage? AEndovascular repair of the aneurysm should be delayed at least one month BGlucocorticoids should be given to decrease intracranial pressure CHematoma evacuation should be done immediately DNimodipine administration improves outcomes

Nimodipine administration improves outcomes Patients with increased intracranial pressure may require placement of a ventriculostomy for monitoring and management. Supportive care consists of intravenous fluid administration to achieve euvolemia, serum glucose control, prophylactic therapy to prevent gastrointestinal ulcers, blood transfusions to maintain hemoglobin between 8 and 10 mg/dL, anticoagulation reversal, blood pressure control to a goal systolic blood pressure of less than 160 mm Hg, stool softeners, analgesia, nimodipine, and bedrest. Nimodipine is a calcium channel blocker that prevents vessel vasospasm. It should be initiated within the first four days and should be continued for 21 days duration with either oral or nasogastric administration. These interventions help to improve outcomes and prevent complications. Subarachnoid hemorrhage is associated with a high mortality rate. About 10% of patients with aneurysmal subarachnoid hemorrhage die prior to reaching the hospital, 25% die within 24 hours, and 45% die within 30 days. Long-term neurologic complications may include neurocognitive dysfunction, epilepsy, and focal neurologic deficits, such as hemiparesis and hemianopsia. Furthermore, individuals who survive have a small but enduring risk for recurrent subarachnoid hemorrhage.

Which of the following studies should always be done prior to administering alteplase in a patient with a suspected ischemic stroke? ACoagulation studies BElectrocardiogram CNoncontrast computed tomography of the head DTroponin

Noncontrast computed tomography of the head Ischemic strokes occur when the arteries to the brain become severely narrowed or blocked leading to deficient cerebral blood flow. Ischemic strokes account for about 85% of strokes, while hemorrhagic strokesaccount for about 15%. Ischemic strokes can be thrombotic or embolic. Thrombotic ischemic strokes are due to atherosclerosis and clot formation in the cerebral vasculature. They are usually caused by plaque buildup that occurs over time. Embolic ischemic strokes occur when a clot forms outside of the cerebral vasculature, such as in the chambers of the heart or in the carotid artery, and travels to the cerebral vasculature where it causes ischemia. Risk factors for an ischemic stroke are similar to risk factors for other cardiovascular disease and include advanced age, male sex, smoking, hypertension, dyslipidemia, diabetes mellitus, and atrial fibrillation. The symptoms and signs of an acute ischemic stroke vary depending on the blood vessel involvedand severity of the ischemia. Common findings include contralateral sensory or motor loss, homonymous hemianopsia (visual field loss on the left or right side of each eye), aphasia (loss of the ability to understand or produce speech), apraxia (inability to perform learned motor movements despite understanding the command), spatial neglect, impaired judgment, and personality changes. Strokes involving the vertebral or basilar arteries may present with vertigo, nystagmus, nausea, vomiting, diplopia, cranial nerve palsies, and cerebellar dysfunction. If an acute ischemic stroke is suspected, then diagnostic testing is needed to rule outalternate diagnoses or a hemorrhagic stroke prior to potential treatment for an ischemic stroke. Noncontrast computed tomography (CT) scan of the head is the initial neuroimaging study to rule out a hemorrhagic stroke. Fingerstick blood glucose should also be obtained immediately because hypoglycemia is a common stroke mimicker. In most patients, additional testing should include an electrocardiogram (ECG), troponin, coagulation studies (e.g., prothrombin time, international normalized ratio, and activated partial thromboplastin time), renal function studies, and electrolytes. The management of ischemic stroke is dependent on the amount of time from the patient's initial symptoms to the medical presentation. Thrombolytic therapy with tissue plasminogen activator (tPA), such as alteplase, can be performed if the patient has no evidence of hemorrhage, presents within four and a half hours of symptom onset, and does not have a contraindicationto tissue plasminogen activator therapy. Absolute contraindications to tissue plasminogen activator therapy include evidence of current intracranial hemorrhage or subarachnoid hemorrhage, active internal bleeding, intracranial trauma or surgery within the past three months, bleeding diathesis, and current severe uncontrolled hypertension. Blood pressure management is also important in patients with acute ischemic stroke and elevated blood pressure. Patients who qualify for tissue plasminogen activator therapy should have their blood pressure lowered to below 185/110 mm Hg. Patients who are not eligible for tissue plasminogen activator therapy, including patients presenting more than four and a half hours after symptom onset, may be eligible for mechanical thrombectomy, which can be performed in the first 24 hours after symptom onset. Secondary prevention is also important to prevent future strokes. Smoking cessation should be emphasized. Hypertension and diabetes mellitus should also be controlled in patients with these comorbidities. Statins, such as atorvastatin, are often used to improve the lipid profile. The recommended dose of atorvastatin is 80 mgdaily. Patients should also be treated with antiplatelet therapy, such as with aspirin. Other lifestyle modifications include reduction of alcohol consumption in heavy drinkers, regular aerobic exercise, weight reduction in individuals with elevated body mass index, and a diet that emphasizes the intake of fruit, vegetables, and whole grains.

A 65-year-old man with a history of hypertension, sleep apnea, and heart failure presents to his primary care provider with a complaint of having a difficult time walking for the past four months that is progressively getting worse. He states that he has had urinary urgency that started around the same time that has progressed to occasional incontinence. His wife states that he has had difficulty concentrating and seems to not care about things. He denies nausea, vomiting, headaches, and vision changes. On physical exam, there is no papilledema or nuchal rigidity. Cognitive changes are noted. His gait is slow with small steps and a wide base. He loses his balance when he turns. Laboratory results are normal. Magnetic resonance imaging demonstrates ventriculomegaly and a disproportionate enlargement of the Sylvian fissures. Which of the following is the most likely diagnosis? AAlzheimer disease BLewy body dementia CNormal pressure hydrocephalus DParkinson disease

Normal pressure hydrocephalus Normal pressure hydrocephalus is a condition that is caused by the enlargement of the ventricles in the brain and an accumulation of cerebrospinal fluid. A lumbar puncture would show no elevation in pressure due to the ventricle enlargement allowing for accumulation without increasing pressure. The condition is seen in theelderly with those most at risk being over 80 years of age. Hypertension or a decrease in the absorption of the cerebral spinal fluid may cause the condition. Secondary causes include chronic meningitis and subarachnoid hemorrhage. The classic triad of symptoms of normal pressure hydrocephalus include gait apraxia, urinary incontinence, and cognitive difficulty. Diagnosis is made after excluding other causes of the symptoms through laboratory testing and imaging. Magnetic resonance imaging is superior to computed tomography in evaluating ventricle and sulcus size. A disproportionately enlarged subarachnoid space would suggest the condition. Imaging may also indicate how successful treatment outcomes may be. Other testing, such as high-volume lumbar puncture or lumbar drain trial, may be used to confirm the diagnosis. Treatment of normal pressure hydrocephalus is an implanted ventricular shunt. Due to possible complications following shunt placement, regular follow-up with attention to symptoms is recommended.

Which of the following physical findings is most suggestive of infectious tenosynovitis involving a finger flexor tendon sheath? AAbducted thumb and swelling over the first web space BLymphangitic streaking extending to the forearm CPain with passive extension of the finger DSwelling and tenderness of the distal pulp of the finger

Pain with passive extension of the finger Infectious or suppurative flexor tenosynovitis is a bacterial infection of a tendon sheath. Most infections occur through direct inoculation of the tendon sheath with bacteria but can also occur by spread from contiguous structures and less commonly by hematogenous seeding. Common causes of direct infection are human and animal bite wounds and puncture wounds. The diagnosis is primarily clinical. Kanavel described four signs that have been shown to be highly sensitive: pain with passive extension of the finger, flexed posture of the finger at rest, fusiform swelling of the digit, and tenderness along the flexor sheath. Pain with passive extension is typically seen early in the course of infection and is an important finding. Surgical drainage and irrigation andintravenous antibiotics are required for most infections, although sometimes early infections (less than 48 hours) can be treated with splinting, elevation, and antibiotics. Common complications include stiffness, tendon necrosis, boutonniere deformity, and recurrence of infection.

A 45-year-old man presents to your office with a complaint of worsening asthma symptoms. He tells you that he has had asthma since he was a child and is compliant with his daily inhaled glucocorticoid that has controlled his asthma until recently. He also reports recent sinus pressure. Labwork results show eosinophilia of more than 10% in a peripheral blood smear. To confirm the diagnosis of eosinophilic granulomatosis with polyangiitis, which of the following also needs to be present? AHypotension BParanasal sinusitis CVertigo DVomiting

Paranasal sinusitis Eosinophilic granulomatosis with polyangiitis is a rare systemic disorder that affects small-to-medium-sized vessels. Necrotizing vasculitis of these vessels results in severe asthma, chronic rhinosinusitis, and both blood and tissue eosinophilia. Also referred to as Churg-Strauss syndrome, it affects men and women equally. The average age of onset is 38 years and the average age of diagnosis is 50 years. There are three stages of the disease. The first stage involves asthma and allergic rhinitis, with asthma being the cardinal feature of the disease, affecting more than 95% of patients. Eosinophilic granulomatosis with polyangiitis should be considered in individuals who are compliant on a moderate dose of inhaled glucocorticoids but have poorly controlled asthma. Worsening of asthma symptoms often leads to the need for longer courses of oral glucocorticoids to control the symptoms and may inhibit diagnosis due to masking of symptoms. Paranasal sinusitis is also a part of the first phase of eosinophilic granulomatosis with polyangiitis, which affects more than 60% of individuals with the disorder. The second phase is called the eosinophilic phase. Features of this phase include eosinophilic infiltrates in multiple organs, especially the lungs and gastrointestinal tract, along with peripheral blood eosinophilia. The third phase is the vasculitic phase of the disorder, which is life-threatening. This phase generally develops approximately three years after the first phase, although it may be delayed for longer than this, even up to several decades. Symptoms of this phase are related to a number of systems and can include pulmonary, dermatologic, cardiac, renal, and peripheral nerve involvement. Dermatologic complications range from palpable purpura to subcutaneous nodules with skin biopsy confirming the diagnosis. In addition to the peripheral neuropathy, neurologic complications also include central nervous system manifestations such as subarachnoid and cerebral hemorrhage, cranial nerve palsies, and visual loss. Cardiac complications are some of the most serious seen in eosinophilic granulomatosis with polyangiitis with approximately half of the deaths due to the disorder being related to myocardial infarction or myocarditis. Diagnosis typically begins with clinical suspicionbased on symptoms. Confirmation of the diagnosis can be difficult due to manifestations of the disorder occurring in isolation from one another and there not being a lab test that is specific for eosinophilic granulomatosis with polyangiitis. The hallmark laboratory finding is eosinophilia. Other lab findings in patients with the disease include elevated erythrocyte sedimentation rate, C-reactive protein, and IgE, along with positive antineutrophil cytoplasmic antibodies and rheumatoid factor. Treatment is with glucocorticoids.

A 70-year-old man presents to the clinic with hand shakiness and a lack of coordination for the past several months. He says that his right hand shakes while at rest. Neurologic examination reveals cogwheel rigidity. Which of the following is the most likely diagnosis? ADementia with Lewy bodies BEssential tremor CNormal pressure hydrocephalus DParkinson disease

Parkinson disease

A 35-year-old woman presents to your office with a three-month history of foot pain. She is an avid runner. Examination is suggestive of a Morton neuroma. Which of the following recommendations is appropriate for initial treatment of her condition? AChanging to a shoe with a narrow toe box BInjecting a glucocorticoid into the intermetatarsal space CPlacing a pad in her shoe just proximal to the metatarsal heads DUsing kinesiotape to strap the arch of her foot

Placing a pad in her shoe just proximal to the metatarsal heads Changing to a shoe with a narrow toe box (A) may exacerbate the patient's symptoms, as Morton neuromas are associated with narrow footwear. Injection of a glucocorticoid into the intermetatarsal space (B) is not an appropriate initial treatment. Injections are effective for symptoms that are not improving with shoe modifications but can cause atrophy of the metatarsal fat pad. Using kinesiotape to strap the arch of her foot (D) is unlikely to help the patient's symptoms. Strapping may be helpful for symptoms of posterior tibial tendon dysfunction.

A 57-year-old woman presents to your office with four months of right heel pain. She states that the pain is much worse when she gets up in the morning. She is a runner, and she has startup pain with running that improves after about a half mile. On examination, she is tender over the plantar aspect of the calcaneus. Her Achilles tendons are tight bilaterally with ankle dorsiflexion of only a few degrees with the knees in extension. Which of the following is the most likely diagnosis? AAchilles tendinitis BPlantar fasciitis CRetrocalcaneal bursitis DStress fracture of the calcaneus

Plantar fasciitis

A 40-year-old man presents to the clinic with about two months of generalized fatigue and occasional fevers. He also reports unexplained weight loss of about 15 pounds in that time. Last week he started noticing more joint pain in his shoulders, hips, and knees and noticed some skin discoloration on his legs. Upon physical exam, the patient is afebrile but has a lacy, purplish-red rash on his lower extremities bilaterally that is non-tender to palpation and some ulcerations of the left ankle around the medial malleoli. You order a tissue biopsy of the area surrounding the ulcer. Which of the following is the most likely diagnosis? AErythema nodosum BPeripheral vascular disease CPolyarteritis nodosa DPolymyositis

Polyarteritis nodosa Polyarteritis nodosa (PAN) is a systemic necrotizing vasculitis that causes inflammation of the medium-sized vessels and can involve any organ but commonly involves the skin, peripheral nerves, mesenteric vessels, kidneys, heart, and brain. Most cases are idiopathic, however, there is a known association withhepatitis B (HBV) and hepatitis C (HCV) viruses. Polyarteritis nodosa is most common in middle-aged andolder adults, with peak incidence in the sixth decade of life, and has a male predominance. Patients present with generalized symptoms of fatigue, fever, and weight loss that has an insidious onset developing over weeks to months. Patients may then develop joint pain, myalgia, or neuropathy. Clinical features depend on the arteries involved. Skin findings include livedo reticularis (lacy, purplish discoloration of the skin),subcutaneous nodules, and skin ulcers that are a result of underlying vasculitis of the medium-sized vessels. The most common skin finding is lower extremity ulcerations, usually near the malleoli. Patients may also have renal hypertension if the renal arteries are involved. Patients with mesenteric vessel involvement may haveabdominal pain, nausea, and vomiting. Those with cardiac involvement may develop cardiac dysfunction. There are no specific laboratory tests to diagnose polyarteritis nodosa, although patients may commonly be anemic with leukocytosis. Patients are classically antineutrophil cytoplasmic antibody (ANCA)-negative, may have elevated acute phase reactants, and have low titers of rheumatoid factor and antinuclear antibodies. All patients suspected of this condition should be tested for hepatitis with HBV and HCV serologies. Diagnosis can be confirmed by tissue biopsy of symptomatic sites (e.g., a skin nodule) or angiogram (which may show dilations or aneurysms in the affected arteries). Treatment with high dose corticosteroids is the mainstay of treatment for patients with relatively mild disease. Immunosuppressive agents such as azathioprine, methotrexate, and cyclophosphamide, are reserved for patients with moderate to severe disease.

A 50-year-old woman presents with a new onset of severe headaches. She has a 30 pack-year history and consumes excessive amounts of alcohol. Past medical history is significant for hypertension and polycystic kidney disease. Which of the following poses the greatest risk for the development of a cerebral aneurysm in this patient? AAlcohol consumption BCigarette smoking CHypertension DPolycystic kidney disease

Polycystic kidney disease A cerebral aneurysm (a saccular or a berry aneurysm) occurs at an arterial bifurcation. It is most commonlyfound on the anterior part of the circle of Willis. An unruptured aneurysm is usually asymptomatic. A subset of patients with unruptured aneurysms may present with symptoms of a headache, visual disturbances, facial pain, and third nerve palsy. Subarachnoid hemorrhage results when a cerebral aneurysm ruptures. Additional symptoms include nausea, vomiting, meningismus, seizures, brief loss of consciousness. Patients with subarachnoid hemorrhage classically present with an abrupt onset of a severe headache described as "the worst headache of their lives." A cerebral aneurysm affects approximately 3.2% of the population. Men and women are equally affected. The incidence of subarachnoid hemorrhage is approximately six to 16 per 100,000 per year. Risk factors for a cerebral aneurysm include polycystic kidney disease, connective tissue disease, coarctation of the aorta, familial aldosteronism type 1, cigarette smoking, hypertension, and alcohol consumption. Polycystic kidney disease confers the highest risk (6.9 times) of developing a cerebral aneurysm. Computed tomography angiography or magnetic resonance angiography can detect aneurysms that are 5 mm or larger with great sensitivity. Cerebral angiography is the diagnostic tool of choice when aneurysms are 2 mm or less. For subarachnoid hemorrhage, non contrast CT scan is the gold standardespecially within the first 24 hours. If the CT scan is negative with high clinical suspicion, lumbar punctureis mandatory. Results of the lumbar puncture will reveal high opening pressure and xanthochromia that does not diminish from tube one to four. Treatment is either by surgical clipping or endovascular coiling. The patient must be managed in an intensive care unit for medical, neurological, and hemodynamic monitoring. Postoperatively, treatment with nimodipine for 21 days prevents vasospasm and improves neurologic outcomes. For patients with elevated intracranial pressure, ventriculostomy is recommended. Complicationsof subarachnoid hemorrhage include vasospasm, rebleeding, hydrocephalus, seizures, increased intracranial pressure. The prognosis for subarachnoid hemorrhage depends on age at presentation, clinical or radiologic severity of the subarachnoid hemorrhage, and occurrence of complications. Unfortunately, subarachnoid hemorrhage is associated with a high mortality rate and long-term neurologic complications including neurocognitive dysfunction, epilepsy, and other focal neurologic deficits.

A 55-year-old woman presents to the clinic with several months of fatigue, bilateral shoulder pain, and bilateral hip pain. She states that she is achy and stiff in those areas, especially when she gets up in the morning. Only after lying in bed for about an hour can she get up and walk around. She does not complain of any headaches or scalp tenderness. Laboratory tests reveal an erythrocyte sedimentation rate of 80 mm/hr and a normal creatine kinase. Which of the following is the most likely diagnosis? AFibromyalgia BGiant cell arteritis CPolymyalgia rheumatica DPolymyositis

Polymyalgia rheumatica Polymyalgia rheumatica is a chronic inflammatory disease of unknown cause that primarily affects elderly persons. It is characterized by morning stiffness and pain in the proximal muscle groups, particularly in thehips and shoulders, that lasts for more than one hour. It is also closely associated with giant cell arteritis(also known as temporal arteritis). It is common in those of northern European descent and is twice as common in women as in men. It primarily affects persons aged 50 years or older, and the incidence increases with age. Patients are generally in good health when they present with symptoms of polymyalgia rheumatica. Initial symptoms of pain and stiffness are most commonly found in the shoulder girdle bilaterally as well as the pelvic girdle, hips, and neck. Stiffness may be severe, especially after periods of rest and in the morning, and will improve after about one hour. Muscle strength is not affected. Generalized findings that are common include low-grade fever, weight loss, malaise, fatigue, and depression. Some features include synovitis of proximal joints, peripheral arthritis, carpal tunnel syndrome, and swelling of the extremities. Physical exam findings arenonspecific due to the variety of presenting symptoms. Polymyalgia rheumatica is diagnosed clinically and is a diagnosis of exclusion. There are several diagnostic criteria for this condition, and one set includes age of onset of 50 years or older, erythrocyte sedimentation rate ≥ 40 mm/hr, pain persisting for more than one month and involving at least two areas (the neck, shoulders, and pelvic girdle), morning stiffness lasting more than one hour, rapid response to prednisone therapy, all in the absence of other diseases. Although there is no specific test for this condition, it is recommended that certain laboratory studies be performed to rule out other possible diseases and to establish baseline values to monitor therapy. Such tests include rheumatoid factor, anticyclic citrullinated peptide antibodies, C-reactive protein (which will be elevated), ESR (which will be elevated), complete blood count with differential (may reveal mild anemia with slightly elevated white blood cell and platelet counts), blood glucose, serum creatinine, liver function tests (normal), calcium levels, alkaline phosphatase levels (may be elevated), urinalysis, thyroid-stimulating hormone, creatine kinase (which will be normal), and vitamin D testing. ESR is sensitive in polymyalgia rheumatica but not specific since it is not always elevated in patients with this condition. Radiographs of the joints will be normal or show some osteoarthritis. Magnetic resonance imaging of the shoulder will reveal areas of bursitis and synovitis. Polymyalgia rheumatica is a chronic, self-limited disorder (lasting from several months to five years), whose treatment is based on symptomatic treatment of muscle pain, stiffness, and generalized symptoms (e.g., fever and fatigue). Corticosteroids (e.g., prednisone) are the treatment of choice, and patients will have rapid improvement of symptoms when treated with low doses. Dosage and duration of treatment depends on the patient's weight and symptom severity. Effective treatment is also shown in the reduction of the ESR to normal. Nonsteroidal anti-inflammatories may also provide pain relief. In those in whom corticosteroids are contraindicated, drugs such as methotrexate, tocilizumab (an interleukin-6 receptor antagonist), and tumor necrosis factor alpha-inhibitors may also be effective. Patients have an excellent prognosis, but relapse is common, therefore, corticosteroid treatment is usually prescribed for at least one to two years, with some patients requiring treatment for longer periods. All patients with this condition should be monitored regularly for giant cell arteritis development.

A 70-year-old man presents to the clinic with bilateral joint pain and morning stiffness. His shoulders and hips have the most prominent symptoms. The symptoms began abruptly three days ago. He denies any history of trauma. Physical examination reveals a decrease in range of motion but normal strength. Laboratory findings include an elevated erythrocyte sedimentation rate. Which of the following is the most likely diagnosis? AFibromyalgia BOsteoarthritis CPolymyalgia rheumatica DRheumatoid arthritis

Polymyalgia rheumatica Polymyalgia rheumatica is an inflammatory rheumatologic condition characterized clinically by aching pain and morning stiffness of the shoulders, hips, neck, and torso. It is often associated with giant cell arteritis. Polymyalgia rheumatica almost exclusively affects adults over the age of 50. The incidence increases progressively with advancing age, and the average age at the time of diagnosis is over 70 years old. Patients present with symmetrical aching and stiffness of the proximal extremities. The symptoms are often most severe in the morning, and the onset is usually recent and abrupt. A presentation of longstanding stiffness and aching is atypical and suggests an alternative diagnosis. Patients may complain of difficulty combing their hair, putting on a coat, and rising from a chair. Patients with polymyalgia rheumatica may also experience nonsevere systemic symptoms such as malaise, fatigue, anorexia, weight loss, and low-grade fever. Physical examination findings may include limited range of motion, but strength testing is usually normal. The most common laboratory finding is an elevated erythrocyte sedimentation rate (ESR). Serologic tests, such as antinuclear antibody, rheumatoid factor, and anticyclic citrullinated peptide, are all normal. Creatine kinase is also normal since the disease process targets the synovium and bursae and not muscles themselves. Polymyalgia rheumatica is a clinical diagnosis. Age above 50 years at disease onset, along with proximal and bilateral aching and morning stiffness, suggests the diagnosis. An elevated erythrocyte sedimentation rate is also supportive, although a normal value does not exclude the diagnosis. The response to low-dose glucocorticoids helps to confirm the diagnosis. Patients with polymyalgia rheumatica usually have significant response to glucocorticoids within three days and complete resolution of symptoms within three weeks. Low-dose glucocorticoids, such as prednisone, are the treatment of choice. Glucocorticoids are dosed to suppress symptoms and then gradually tapered over months. All patients with polymyalgia rheumatica must be monitored for the development of giant cell arteritis, and if symptoms are present, a temporal artery biopsy must be performed.

Which of the following physical exam findings would be most consistent with an anterior cruciate ligament tear? APositive anterior drawer test BPositive Apley grind test CPositive McMurray test DPositive Thessaly test

Positive anterior drawer test

What is the best treatment for a patient with severe Bells Palsy? ACarbamazepine and valacyclovir BPrednisone and valacyclovir CPrednisone monotherapy DValacyclovir monotherapy

Prednisone and valacyclovir Bells palsy is defined as an acute, unilateral peripheral facial nerve (cranial nerve VII) palsy of unknown cause. There is no predilection based on gender or race, but the risk is three times greater during the third trimester of pregnancy or the first week postpartum. Herpes simplex virus activation is likely the cause of Bells palsy in most cases. However, since there is no established method to confirm herpes simplex virus as the mechanism, it is labeled as Bells palsy in clinical practice. Bells palsy usually presents with an abrupt onset of unilateral facial paralysis. Common findings include disappearance of the nasolabial fold, inability to close the eyes, and the mouth appearing drawn to the unaffected side. Associated manifestations may include decreased tearing, hyperacusis (increased sensitivity to sounds), and loss of taste sensation on the anterior two-thirds of the tongue. The forehead lift examination can help in differentiating between a peripheral and central cause of a facial muscle palsy. Each side of the forehead has bilateral innervation from the central nervous system. Therefore, central lesions (strokes) typically spare the forehead. In contrast, Bells palsy does not usually spare the forehead. Patients can be asked to raise their eyebrow to look for sparing of the forehead. Intact motor movement of the forehead suggests that the etiology may be central. However, this finding is simply supportive and is not diagnostic. The diagnosis of Bells palsy is based on clinical criteria. The clinical criteria include diffuse facial nerve involvement manifesting as paralysis of the facial muscles. The symptoms must present abruptly (over one or two days), reach maximal clinical weakness within three weeks, and show some degree of improvement within six months. Neuroimaging is usually not needed initially in patients with Bells palsy. However, if the clinical course becomes atypical, then contrast-enhanced neuroimaging should be obtained. The clinical course is considered atypical if there is slow progression beyond three weeks or no improvement at four months. In these cases, neuroimaging should be obtained to rule out a tumor. Serologic testing for Lyme disease should be ordered at the initial presentation if there is bilateral facial nerve involvement. All patients with Bells palsy should be treated with oral glucocorticoids, such as prednisone, for one week. Glucocorticoids are most effective when initiated within three days of symptom onset. Patients with severe symptoms, such as disfiguring asymmetry or incomplete eye closure with maximal effort, should also be treated with valacycloviralso. Patients with poor eyelid closure and reduced tearing should be treated with artificial tears and eye protection, such as glasses. Severity is determined by the House-Brackmann classification of facial nerve dysfunction, which is based on the amount of movement of the forehead, mouth, and eye. Patients with incomplete recovery may try physical therapy. Surgical decompression is another theoretical treatment, but it is not recommended because there is no evidence that it works. Botulinum toxin injections may benefit patients with facial spasm or hyperlacrimation. Brow ptosis correction may enhance facial symmetry and cosmeticappearance. Furthermore, weight insertion into the upper eyelid can improve eye closure.

A 48-year-old man presents to your office with a one-week history of right knee swelling. He works installing hardwood floors and finds it difficult to kneel. On physical examination, he has full range of motion of the knee. He has no effusion in the knee but has a 3-centimeter, well-circumscribed, subcutaneous, nontender fluid collection over the patellar tendon. Which of the following is the most likely diagnosis? AAcute gout BInflammatory arthritis CMeniscal cyst DPrepatellar bursitis

Prepatellar bursitis

A 26-year-old woman with a history of poorly controlled asthma presents to the office complaining of worsening migraine headaches. She has been using sumatriptan 100 mg orally at the onset of headache for abortive relief but finds she is running out of the prescribed number of pills two weeks into each month. Which of the following medications would be the best therapy at this time to prevent her migraines altogether? AErenumab BPropranolol CTopiramate DValproic acid

Propranolol Topiramate is an appropriate preventative treatment for migraine headaches. Migraine headache is a common neurological disorder characterized by episodic, severe headaches associated with nausea, light sensitivity, sound sensitivity, and, sometimes, aura. Migraines occur secondary to spreading cortical depression, primary neuronal dysfunction, and activation of the trigeminovascular system. It is often inherited. It is more common in women. Migraines have four phases: prodrome, aura, headache, and postdrome. Prodromal symptoms occur 24-48 hours before headache onset and can include yawning, food cravings, mood changes, constipation, and muscle stiffness. Migraine with aura (classic migraine) occurs in approximately one-fourth of patients with the headache disorder. An aura is a focal reversible neurological symptom lasting no more than one hour immediately prior to or coinciding with the start of the headache. Aura symptoms can include visual disturbances or occult vision loss, paresthesia and other sensory disturbances, dysphasia, and motor disturbances. A patient with motor aura, primarily weakness of the face or limb, is classified as having a hemiplegic migraine, which is a rare migraine variant. The migraine headache is often unilateral with increasing throbbing or pulsatile intensity. The pain, when untreated, can generally last for four hours to several days. During the postdrome period, patients often experience fatigue. A variety of factors can trigger migraines, including hormonal changes, stress, barometric pressure changes, offending odors, alcohol, and certain foods. Diagnosis is based on patient history and physical examination. For adequate diagnosis, patients should have a history of at least five headache attacks with migrainous characteristics lasting 4 to 72 hours. Imaging is not necessary unless there is an unexplained abnormal finding on neurological examination or atypical headache features. Preventative migraine treatment should be considered in patients with frequent attacks (4 or more headaches per month or headaches that last longer than 12 hours), those who do not respond to acute therapies, or those with risk of medication overuse headache. Initial therapy can include a tricyclic antidepressant (amitriptyline), beta-blocker (propranolol), or anticonvulsant (topiramate), depending on the patient's history and comorbid conditions. For mild to moderate migraines, simple analgesics, such as naproxen, can be used for acute management. For moderate to severe attacks, oral migraine-specific agents are first line as long as there are no contraindications. These include triptans and ergot derivatives.

A 40-year-old woman presents to the clinic with pain and morning stiffness in her fingers and toes. She describes the morning stiffness as lasting more than 30 minutes but says it improves throughout the day. On physical examination, you note a "sausage appearance" of her fingers and toes along with joint line tenderness and effusion of the distal interphalangeal joints. You also note onycholysis and nail pitting. There is an erythematous, thick plaque with silvery scale noted around her umbilicus. Laboratory results reveal an elevated erythrocyte sedimentation rate and uric acid levels, however, rheumatoid factor and anti-cyclic citrullinated peptide antibodies are absent. The synovial fluid examination does not show any monosodium urate crystals. Radiography of the fingers and toes shows a "pencil-in-cup" deformity. Which of the following is the most likely diagnosis? AGout BPsoriatic arthritis CReactive arthritis DRheumatoid arthritis

Psoriatic arthritis Psoriatic arthritis is seronegative inflammatory arthritis that follows psoriasis in more than 80% of cases. Psoriatic arthritis was formerly considered a variant of rheumatoid arthritis but is now considered a separate disease. There is no predilection to sex as it affects both men and women equally with an incidence of six per 100,000 annually. Several subsets of psoriatic arthritis have been described. Several of these subsets include symmetric polyarthritis that mimics rheumatoid arthritis; a destructive oligoarticular form; arthritis mutilansmarked by osteolysis; a monoarticular form that affects the distal interphalangeal and causes nail pitting and onycholysis; and a spondylotic form that predominantly affects the sacroiliac joint and spine. Clinical manifestations of psoriatic arthritis usually depend on which subset the patient has. The patient in the vignette above presents with asymmetric pain and morning stiffness in the distal interphalangeal and sacroiliac joints. The morning stiffness usually lasts more than 30 minutes and improves with activity. About 70% of patientswill have a history of psoriasis, although some patients are unaware that they have it. Hence, a thorough search for cutaneous patches is pertinent in patients with new-onset arthritis. Nail pitting and "sausage-like"appearance of the digits are also good clinical indicators of the disease. Diagnosis is made by both laboratory studies and imaging. Labs show an elevated erythrocyte sedimentation rate along with uric acid levels, although these patients do not have a predisposition to gout any more than the general population. Rheumatoid factor and anticyclic citrullinated peptide antibodies are also not present. Radiographic evidence of psoriatic arthritis includes marginal osteolysis, pencil-in-cup deformity, atypical syndesmophytes, and asymmetric sacroiliitis. Treatment of psoriatic arthritis follows a specific order. First-line treatment is nonsteroidal anti-inflammatory drugs followed by methotrexate if first-line treatment is not effective. For cases refractory to methotrexate, tumor necrosis factor inhibitors, such as ustekinumab and secukinumab, are used. Tumor necrosis factor inhibitors can treat both the skin lesions and arthritis.

A 45-year-old man presents to the emergency department six hours after being involved in a construction accident. He fell through the floor of a building that was being demolished, and his lower body was trapped under a heavy beam. It took several hours to extricate him, and he is complaining of severe pain in both legs. X-rays are negative for fractures. You are concerned about rhabdomyolysis. Which of the following diagnostic tests is most useful for making the diagnosis? ASerum creatine kinase BSerum electrolytes CUrine hemoglobin DUrine myoglobin

Serum creatine kinase Rhabdomyolysis is a syndrome resulting from muscle necrosis and the release of muscle proteins into circulation. Trauma resulting in crush injury or compression of muscles is a common cause. Other causes include muscle injury due to overexertion (such as training for a marathon)or to drugs (statins, niacin, fibrates) or toxins. Patients present with muscle pain, weakness, and dark urine due to myoglobin excretion. The diagnosis is made by measurement of creatine kinase, a protein released from damaged muscle cells. Serum creatine kinase levels of the MM fraction, found in skeletal muscle, will be markedly elevated at 10,000 to 25,000 international units per liter (normal range is 22 to 198). Consequences of rhabdomyolysis include volume depletion from influx of fluid into the injured muscles, electrolyte abnormalities, and metabolic acidosis. Acute kidney injury, sometimes requiring dialysis, is a frequent complication due to volume depletion and toxicity from the heme pigment in myoglobin, which forms casts in renal tubules. Treatment is aggressive fluid resuscitation to improve renal perfusion and increase urine flow. This fluid resuscitation limits cast formation, washes out intratubular casts, and increases urinary potassium excretion. Bicarbonate, to raise the urine pH and decrease the toxicity of myoglobin, may be given in patients with severe rhabdomyolysis, an arterial pH of <7.5, and a serum bicarbonate of <30 milliequivalents per liter. Mannitol and other diuretics are not indicated for treatment of rhabdomyolysis. Patients who undergo fluid resuscitation need to be monitored for electrolyte abnormalities and for the development of compartment syndrome in the extremities. Compartment syndrome occurs when there is increased pressure in a muscle compartment surrounded by intact fascia. Decreased perfusion and eventual muscle ischemia results. Symptoms of increased pain, especially with passive stretch of the muscles, and physical examination findings of swelling and tightness of the compartment should prompt the clinician to measure compartment pressures and perform fasciotomy if necessary.

Which of the following increases the risk of osteoporosis in postmenopausal women? ACalcium and vitamin D supplementation BMinimal alcohol intake CRegular physical activity DSmoking cigarettes

Smoking cigarettes

A 40-year-old woman presents to the emergency department after sustaining an injury in a basketball game. She thinks someone might have run into her ankle when she was going up for a jump. She is generally healthy and actively participates in sports, including a basketball team. She has had previous urinary tract infections treated with ciprofloxacin, the last occurrence was four months ago. On exam, calf squeeze is positive indicating likely Achilles tendon rupture. Which of the following is the most important risk factor for this patient's injury? AAge BFemale sex CFluoroquinolone use DSports participation

Sports participation Achilles tendon injury and rupture most frequently occur during recreational sport activities. Male sex, increasing age, and obesity increase the risk for injury, particularly in the context of athletic endeavors. The Achilles tendon is the largest tendon in the body and is instrumental in the body's ability to ambulate, run, jump, accelerate, and decelerate. Overuse, strain, and injury, whether by mechanical or organic causes, compromise its integrity and elasticity. Medications such as fluoroquinolone antibiotics and glucocorticoids are associated with Achilles tendinopathy, but these cases are rare compared to the more common causes of strain and overuse. The most common site of injury is 2 to 6 cm above its insertion point where perfusion is poorest andhealing from microtrauma is slowest. Ruptures occur when sudden, shear stresses are applied to an already weakened Achilles tendon. Patients often report a "pop" or feeling of being struck in the back of the ankle. Pain and inability to ambulate may or may not accompany this injury. On physical exam, an Achilles tendon defect can usually be appreciated on palpation, and the Thompson test, also known as the "calf squeeze" test, detects Achilles tendon rupture when it fails to elicit plantar flexion. Imaging is usually not needed for diagnosis, but ultrasound can be used for confirmation. Initial management includes supportive care, such as ice, pain relief via medication like NSAIDs, reset, and immobilization with the ankle in plantar flexion. All complete Achilles tendon ruptures should be referred for orthopedic evaluation; surgery is usually required, and preventive measures such as dynamic stretching before activity and maintaining a healthy body weight are advised to prevent new or recurrent achilles tendon injury. Age (A) can be a risk factor for Achilles tendon rupture, with increasing incidence as age increases. The peak age for this injury is 30-40 years old. Female sex (B) is not a significant risk factor for Achilles tendon rupture. Males are four to five times more likely to have this injury. Fluoroquinolone use (C) is only rarely associated with Achilles tendinopathy, injury, or rupture, particularly when last exposure to fluoroquinolones has exceeded 90 days.

Which of the following pathogens is the most common microbial cause of osteomyelitis? AEscherichia coli BMycobacteria tuberculosis CStaphylococcus aureus DStreptococcus pyogenes

Staphylococcus aureus Osteomyelitis is an infection of a bone. Osteomyelitis can occur as a result of contiguous (direct) spread of infection from adjacent soft tissues or joints to a bone, hematogenous seeding (typically due to bacteremia), or by direct inoculation of infection into the bone as a result of trauma or surgery. Hematogenous seeding means spreading from the blood in the setting of bacteremia. Staphylococcus aureus is a common microbial cause of osteomyelitis. Osteomyelitis usually presents with a gradual onset of symptoms over several days. Patients present with dull pain at the involved site with or without movement. Patients may also have local findings, such as tenderness, erythema, warmth, and swelling, or systemic findings, such as fever or rigors. However, patients with osteomyelitis at sites such as the hip, vertebra, or pelvis tend not to have many symptoms other than pain. Patients with diabetes mellitus, open fractures, sepsis, IV drugs use, and decubitus ulcers are at increased risk for osteomyelitis. If a diabetic foot ulcer is larger than 2 by 2 centimeters or bone is visible, then osteomyelitis is highly likely. Laboratory tests are usually nonspecific and therefore not helpful in the diagnosis of acute osteomyelitis. However, leukocytosis and elevation in serum inflammatory markers, such as erythrocyte sedimentation rate (ESR) or C-reactive protein (CRP), may be present. The diagnosis of osteomyelitis is established via culture of bacteria from a bone aspiration. Bone aspiration may not be needed, however, if patients present with supportive clinical findings, positive blood cultures, and radiograph findings consistent with osteomyelitis. Radiographic findings suggestive of early osteomyelitis may include soft tissue swelling, osteopenia, cortical loss, bony destruction, and periosteal reaction. Chronic findings may include reactive sclerosis, sequestra, and involucrum. Plain radiographs are often the initial imaging study to evaluate for osteomyelitis and are more sensitive if symptoms have been present for at least 10 days. Magnetic resonance imaging (MRI) is the most sensitive imaging study for early osteomyelitis. MRI may demonstrate abnormal marrow edema as early as one to five days following onset of infection. Establishing an accurate diagnosis is critical due to the extent of treatment required. Treatment of osteomyelitis often requires both surgical debridement of necrotic material and IV antimicrobial therapy. Antibiotic selection is delayed in stable patients until tissue cultures can be obtained. If cultures cannot be obtained, then broad-spectrum empiric coverage is used. Empiric coverage typically consists of vancomycin and a third- or fourth-generation cephalosporin. This combination provides coverage against methicillin-resistant Staphylococcus aureus (MRSA) and gram-negative organisms. Therapy is typically given parenterally for several weeks. Rifampin may be used as an adjunct agent when osteomyelitis occurs in a patient with prosthetic material.

Which of the following interventions is recommended in all patients for secondary prevention after a transient ischemic attack? AAnticoagulant therapy BAntihyperglycemic therapy CAntihypertensive therapy DStatin therapy

Statin therapy Anticoagulant therapy (A) is recommended in patients with a transient ischemic attack caused by atrial fibrillation. Patients who have had a transient ischemic attack caused by small or large vessel vascular disease are recommended to take antiplatelet medication rather than anticoagulant medication. Antihyperglycemic therapy (B) is only recommended in patients with diabetes mellitus. Diabetes mellitus is a risk factor for transient ischemic attacks and should be managed by maintaining tight glycemic control through dietary modification and pharmacologic therapy. Antihypertensive therapy (C) is only recommended in patients with hypertension. Hypertension is a risk factor for transient ischemic attacks and should be controlled in individuals with hypertension.

Which of the following muscles is affected in patients with torticollis? ABuccinator BMasseter CSternocleidomastoid DTeres minor

Sternocleidomastoid

A 45-year-old man with a 30 pack-year history of smoking presents to the emergency department after losing consciousness while moving a large piece of furniture about an hour ago. He states that he now has a left-sided headache that has gotten worse over the past hour and is now 10 out of 10 on the visual analog scale for pain. He has some nausea and confusion. He states that he had a similar headache a week ago that resolved. On physical exam, he has elevated blood pressure, decreased range of motion on head flexion, and retinal hemorrhages. An electrocardiogram shows ST segment depression, QT interval prolongation, deep symmetric T wave inversions, and prominent U waves. Which of the following is the most likely diagnosis? ACluster headache BMeningitis CMigraine headache DSubarachnoid hemorrhage

Subarachnoid hemorrhage Subarachnoid hemorrhage is a condition caused by a hemorrhage in the subarachnoid space. Most of these hemorrhages are caused by the rupture of an aneurysm, although in a small number of cases, they are caused by vascular malformations or arterial dissections. Many patients will have a sentinel headache caused by a minor hemorrhage six to 20 days prior to the rupture of the aneurysm. Exertion may trigger aneurysm rupture. With the rupture of the aneurysm, there may be a loss of consciousness just prior to the onset of the headache. The headache will reach its maximum intensity within an hour of onset. It is often characterized as a thunderclap headache or worst headache of one's life. Neck stiffness, nausea, and altered mental status may be present. Several hours following a rupture, it is possible to see blood in the cerebral spinal fluid. Seizures may occur within the first 24 hours following the rupture and are a sign of a poor prognosis. On physical exam, retinal hemorrhages may be present. Electrocardiogram changes following hemorrhage include ST segment depression, QT interval prolongation, deep symmetric T wave inversions, and prominent U waves. Left ventricular dysfunction may also develop. The gold standard for diagnosis is a noncontrast computed tomography scan within the first six hours. If this imaging is negative, it should be followed by a lumbar puncture that would show xanthochromia, a yellow or pink tint, in the case of subarachnoid hemorrhage. The mortality rate following the event is high. There is a chance of rebleeding that increases the incidence of mortality. Treatment of the aneurysm is usually through coiling or clipping the aneurysm. Nimodipine, a calcium channel blocker, is often prescribed to prevent secondary vasospasm. Hyponatremia due to syndrome of inappropriate secretion of antidiuretic hormone and hydrocephalus are common complications following subarachnoid hemorrhage. Patients should be aware of the complications that can occur after the event, such as epilepsy and memory loss.

A 75-year-old chronic alcoholic man presents to the emergency department with head trauma after falling off of a roof. The patient has been in a coma since the time of the fall. Computed tomography of the head reveals a crescent-shaped hemorrhage. Which of the following is the most likely diagnosis? AEpidural hematoma BIntracerebral hemorrhage CSubarachnoid hemorrhage DSubdural hematoma

Subdural hematoma Subdural hematoma (SDH) is defined by bleeding in the potential space between the dura and arachnoid membranes. Subdural hematomas are usually caused by tears in the bridging veins, which drain from the surface of the brain into the dural sinuses. However, arterial bleeds account for 20 to 30% of subdural hematomas. Bleeding eventually ceases due to rising intracranial pressure or compression by the blood clot itself. Subdural hematomas can cross suture lines but are typically limited by dural attachments. Head trauma, which is most commonly due to motor vehicle collisions, falls, and assaults, is the most common etiology of subdural hematomas. Patients with significant cerebral atrophy, such as elderly adults, individuals with prior traumatic brain injury (TBI), and individuals with a history of chronic alcohol abuse, are at the highest risk. Patients who take antithrombotics are also at increased risk. Subdural hematomas may present acutely, subacutely, or chronically. Patients presenting with an acute subdural hematoma are in a coma from the time of injury in about 50% of cases, and patients who are not comatose are typically lethargic. However, a minority of patients have a "lucid" interval prior to neurologic deterioration to a coma. Patients with chronic subdural hematomas may present with insidious headaches, lightheadedness, cognitive impairment, apathy, reduced level of consciousness, and occasionally seizures. The diagnosis of a subdural hematoma is confirmed with neuroimaging. Computed tomography of the brain without contrast is the initial imaging used in cases of head trauma that require imaging. The classic computed tomography scan finding in patients with a subdural hematoma is a crescent-shaped lesion. Subdural hematomas may be managed medically or surgically. The management choice depends on the patient's age, neurologic status (according to Glasgow coma scale), progression of symptoms, acuity of head trauma, and imaging findings, such as hematoma thickness and the presence of a midline shift. Patients with an acute subdural hematoma who have evidence of neurologic deterioration since the time of injury and the potential for recovery should undergo hematoma evacuation. In addition, patients with clot thickness ≥ 10 millimeters or midline shift ≥ 5 millimeters on initial brain scan should undergo surgical management with hematoma evacuation. Patients who are managed medicallyshould be in an intensive care unit. Medical management includes intracranial pressure monitoring and serial head computed tomography scans. Anticoagulation should be reversed in all patients on anticoagulants. Patients initially managed medically should undergo surgery if neurologic deterioration occurs and there is potential for recovery. The estimated mortality rate of patients with an acute subdural hematoma requiring surgery is 50%.

Which of the following is the most appropriate treatment for a patient with enteropathic arthritis that has not responded to initial conservative measures? AAcetaminophen BInfliximab CPrednisone DSulfasalazine

Sulfasalazine Acetaminophen (A) is an agent to consider using as part of initial conservative measures with patients who have enteropathic arthritis. Once a patient has failed conservative measures, the use of acetaminophen or nonsteroidal anti-inflammatory agents should be tapered or discontinued, especially if there is worsening of the patient's inflammatory bowel disease symptoms. A three-month trial of a conventional nonbiologic disease-modifying antirheumatic drug, such as sulfasalazine or methotrexate, is the next step in treatment. If the patient fails this treatment, then infliximab (B) or another of the monoclonal antibody tumor necrosis factor inhibitors is the next step. Prednisone (C) is a corticosteroid that is sometimes used in short courses to treat symptoms of pain and inflammation seen in patients with enteropathic arthritis. It is not recommended for long-term use in treating this condition due to side effects seen with long-term steroid use.

Which of the following blood vessels commonly causes compression of the trigeminal nerve root in trigeminal neuralgia? AMiddle cerebral artery BPosterior cerebral artery CSuperior cerebellar artery DSuperior petrosal sinus

Superior cerebellar artery A vascular loop of the superior cerebellar artery compressing the trigeminal nerve accounts for the majority of trigeminal neuralgia cases. Trigeminal neuralgia (tic douloureux) is a facial pain syndrome occurring within a branch of the fifth cranial nerve. It more commonly affects the V2 (maxillary) or V3 (mandibular) branch. Women are more commonly affected than men. It is more common after the age of 50. An aberrant vesselapplying compression to the trigeminal nerve root causes the vast majority of cases. Other causes include tumor(vestibular schwannoma, meningioma), cyst, aneurysm, and arteriovenous malformation. When the disorder occurs in younger patients, demyelinating disease (multiple sclerosis) should be excluded. Trigeminal neuralgia is characterized by paroxysmal, severe stabbing, unilateral pain within the distribution of the trigeminal nerve branch. Pain lasts for only seconds but can occur repeatedly. It is associated with a refractory period. It does not awaken patients from sleep. Autonomic features are associated with attacks within the V1 (ophthalmic) branch. Triggers include eating, talking, brushing teeth, touching the face, cold air or wind, and facial movements. The diagnosis is based on clinical symptoms. Neuroimaging is recommended to rule out secondary causes, such as structural lesion and multiple sclerosis in young patients. Magnetic resonance imaging with and without contrast is preferred. Pharmacologic therapy with carbamazepine is the initial first-line treatment of choice. Other medication options include oxcarbazepine, baclofen, and lamotrigine. Surgical intervention is recommended for patients who are refractory to medical therapy. Potential procedures include microvascular decompression and ablation procedures (rhizotomy, mechanical balloon compression, radiosurgery, and peripheral neurectomy).

A 50-year-old man presents to the outpatient clinic with a nonhealing wound over his right tibia. He struck his shin against a metal bar several months ago. Physical examination shows a 1-centimeter wound over the subcutaneous anterior border of the tibia. There is minimal surrounding erythema and a small amount of serous drainage. Gentle probing with a sterile hemostat reveals that the wound extends to the bone. X-rays show an area of calcification within the bone surrounded by lucency. Which of the following is the most appropriate treatment? AIntravenous broad-spectrum antibiotics BLocalized radiation therapy and antibiotics CSurgery to debride necrotic tissue and antibiotics DTopical antibiotics and local wound care

Surgery to debride necrotic tissue and antibiotics The patient in the vignette has findings consistent with chronic nonhematogenous osteomyelitis. Osteomyelitis is classified as hematogenous, from bacteremia seeding the bone, versus nonhematogenous, occurring by direct inoculation or spread from adjacent tissues, and by duration (acute versus chronic). Chronic osteomyelitis is present for months to years and is characterized by necrotic bone. Bone necrosis occurs due to compression of blood vessels from elevated pressure in the bone marrow. The most common organisms are Staphylococcus aureus, both methicillin-sensitive and methicillin-resistant strains, coagulase-negative staphylococci, and aerobic gram-negative bacilli, but atypical organisms, such as fungi and mycobacteria, may also cause chronic osteomyelitis. Patients may not have the typical findings of pain, swelling, and erythema that are associated with acute infections, or the symptoms may be intermittent. A nonhealing wound may be present, and a sinus tract is pathognomonic of chronic osteomyelitis. Serum white blood cell count is usually normal, and inflammatory markers, such as the erythrocyte sedimentation rate and c-reactive protein, may be high or normal. Wound cultures typically grow skin flora and are not helpful for diagnosis and treatment. X-raysare indicated and if positive, advanced imaging is not necessary. A sequestrum is an area of necrotic bone surrounded by an area of bone resorption that appears on X-rays as an area of dense calcification surrounded by lucency. Treatment for chronic osteomyelitis involves surgical debridement with complete excision of the necrotic bone and soft tissue, as the necrotic bone is avascular and antibiotics given orally or intravenously are not effective. Antibiotic treatment is prescribed based on organisms cultured from the bone. Prolonged antibiotic treatment is not necessary if all infected bone has been removed and the soft tissues are not infected.

A 65-year-old man presents to the office on referral from his primary physician for symptoms of numbness and tingling in his right hand that have been present for over a year. He has been wearing splints at night, which have helped him sleep but do not help his daytime symptoms. He complains of weakness and difficulty holding onto things. He had electrodiagnostic studies showing severe right carpal tunnel syndrome. Which of the following is the most appropriate next step in treatment? AContinued night splinting and occupational therapy BGlucocorticoid injection to the carpal tunnel CSurgery to divide the transverse carpal ligament DUltrasound therapy

Surgery to divide the transverse carpal ligament Continued night splinting and occupational therapy (A) are unlikely to improve the patient's symptoms as he has not had significant improvement with a trial of splinting. A glucocorticoid injection to the carpal tunnel (B) may provide temporary relief of symptoms but is unlikely to give long-term improvement. Ultrasound therapy (D)has not been shown to be effective in treating carpal tunnel syndrome.

A 75-year-old man presents to the emergency department with pain and swelling in his left knee, which has been worsening for the past three days. Physical examination reveals warmth and a moderate effusion in the knee. He holds the knee in 30° of flexion and complains of pain with further flexion. Which of the following is the most useful diagnostic test for septic arthritis? ASerum C-reactive protein level BSerum white blood cell count CSynovial fluid cell count DSynovial fluid Gram stain

Synovial fluid cell count Septic arthritis is a joint infection that is most often caused by hematogenous seeding of a joint with microorganisms but can also result from direct inoculation by trauma or surgery. Most cases are bacterial, but atypical organisms, such as fungi, mycobacteria, or viruses, can also cause infection. The most common organism in children over 3 years old and adults is Staphylococcus aureus. Most patients are otherwise healthy, but patients with immune suppression, hemoglobinopathies, inflammatory arthritis, diabetes, or a history of glucocorticoid injections are at increased risk. Immunocompromised individuals may be infected with atypical organisms. Patients typically present with a joint effusion and swelling around the joint, warmth, tenderness to palpation, and pain with range of motion. The pain tends to worsen over two to five days. Fever is often but not always present. Most septic arthritis is monoarticular, but patients with inflammatory arthritis or sepsis may present with multiple joint involvement. Aspiration of synovial fluid for cell count and cultures is necessary when septic arthritis is suspected. In bacterial arthritis, the synovial fluid white blood cell count is typically greater than 50,000 and often over 100,000 cells per cubic millimeter with over 90% polymorphonuclear leukocytes. There is overlap with inflammatory arthritis, which has white blood cell counts between 20,000 and 80,000 cells per cubic millimeter. Two exceptions are Lyme arthritis and gonococcal arthritis, which present with a lower synovial fluid white blood cell count and negative cultures. If there is clinical suspicion, Lyme serologic testing or genitourinary cultures should be performed. X-rays are indicated to rule out osteomyelitis. Ultrasoundmay be useful for imaging deep joints, but advanced imaging is not necessary for initial diagnosis. Surgery, either open or arthroscopic, is indicated for urgent drainage and irrigation of the joint. Antibiotics are given empirically, pending fluid culture results, to cover Staphylococcus aureus and other possible organisms based on clinical factors. Complications of septic arthritis include destruction of cartilage, osteomyelitis, and sepsis.

A 23-year-old man is transported to the ED by emergency medical services. He was found obtunded in the street, appears intoxicated, and is unable to give a coherent history. He complains loudly of right arm pain, and on exam, the pain seems out of proportion to physical findings, including exquisite pain elicited with passive motion of his right-hand fingers. His right forearm appears pale and feels cool compared to his left forearm. Surgical consult confirms compartment syndrome. Which of the following is the most common underlying cause of this problem? ABurns BInfection CToxin DTrauma

Trauma Fascial membranes encasing muscle groups of the human anatomy effectively form compartments. Because these fasciae are strong and do not expand, increases in the pressure of that space, whether due to decreased capacity or increased volume, compromise perfusion and tissue function within the compartment; this is compartment syndrome. The most common inciting cause of compartment syndrome is acute trauma, especially fracture. The most common sites are the leg and the forearm, although compartment syndrome can occur in other areas as well, including the abdomen. The "6 Ps" describe the typical presentation: pain with passive motion of fingers and toes, paresthesias, pallor, poikilothermia, pulselessness, and paralysis, although the last two are late findings. Young men less than age 35 are most commonly affected. Clinical diagnosis can often be made based on history and exam, but compartment pressures can also be measured by manometers to confirm diagnosis or manage patients. Acute compartment syndrome is a surgical emergencyrequiring immediate fasciotomy to release pressures and restore perfusion. Early identification and treatment usually leads to good functional and cosmetic outcomes. Patients with delayed or missed diagnoses are at risk for long-term complications like infection, fracture nonunion, muscle contractures, sensory deficits, paralysis and even amputation.

Which of the following statements best describes status epilepticus? AFour minutes of continuous seizures BThree discrete seizures between which there is a complete recovery of consciousness CThree minutes of continuous seizures DTwo or more seizures between which there is incomplete recovery of consciousness

Two or more seizures between which there is incomplete recovery of consciousness

According to the Salter-Harris classification system, which type of growth plate fracture involves the metaphysis, physis, and epiphysis? AType I BType II CType III DType IV

Type IV

Which of the following is most consistent with the diagnosis of a febrile seizure? ABacterial illnesses are the predominant cause BMost common in children less than three months of age CMost common in children over five years of age DViral illnesses are the predominant cause

Viral illnesses are the predominant cause Viral illnesses are the predominant cause of febrile seizures. Febrile seizures are the most common type of seizure in the pediatric population and most commonly occur between the ages of three months to five years. Animal studies suggest that endogenous pyrogens during febrile illness may be responsible for increasing neuronal excitability leading to seizures in this population. Febrile seizures can be classified into either simple or complex febrile seizures. Simple febrile seizures present with generalized convulsions, last less than 15 minutes, and do not recur within 24 hours. Complex febrile seizures may present focally, last longer than 15 minutes, and recur over the course of 24 hours. Physical exam generally reveals a fever over 101°F, however, occasionally the fever may spike after the seizure occurs. Examination also reveals the cause of the viral illness such as pharyngitis, otitis media, or a viral exanthem. Serial neurologic examinations should be performed along with tests for meningeal signs. An extensive diagnostic workup is not required for cases of simple febrile seizures where a source of infection is identified and the patient recovers quickly. However, when complex febrile seizures occur and a more serious source of infection is suspected, diagnostic workup may include imaging, laboratory testing, and lumbar puncture. Acute treatment includes airway management and supportive care. For seizures lasting longer than five minutes, at home rectal, buccal, or intranasal diazepam may be administered and continued in the emergency department if necessary. Sources of the febrile illness should be treated and antipyretics may be administered.a

Which of the following physical examination findings is most likely in a patient with a herniated nucleus pulposus at the L4-5 level? AAbsent ankle jerk reflex BAbsent knee jerk reflex CWeakness of ankle dorsiflexion DWeakness of ankle plantar flexion

Weakness of ankle dorsiflexion

A 57-year-old Caucasian man presents to his primary care physician for two days of fever and cough with shortness of breath. A chest X-ray demonstrates a right lower lobe pneumonia. There is also an incidental finding of lytic lesions in the first lumbar vertebral body consistent with Paget disease. Which of the following is the best test for initial diagnosis of this condition? ABone biopsy BCT Scan CRadionuclide bone scan DX-ray

X-ray Paget disease is a disease of the bone resulting in accelerated bone remodeling and growth, very likely due to dysfunction of osteoclasts, which resorb bone. Latent viral infections are also a suspected etiology in genetically susceptible individuals. Paget disease is uncommon in patients less than 55 years old and is associated with European descent. The most common locations of Paget disease are the skull, spine, pelvis, and long bonesof the lower extremities. It is usually asymptomatic and found incidentally. The most common incidental findings are an elevated alkaline phosphatase or lesions noted on plain films taken for another reason. Typical findings on X-ray reflect high bone turnover, including a "mosaic" lamellar bone pattern, "flame-shaped" lesionsdemonstrating disease progress moving in one direction, and lytic and sclerotic lesions. These findings are sufficient for diagnosis, particularly when these radiologic changes are seen at multiple skeletal sites in the context of serum alkaline phosphatase elevation. When symptomatic, it is usually due to the secondary consequences of bone overgrowth, such as bone tumors, fractures, electrolyte abnormalities, nerve impingement, osteoarthritis, or excessive bleeding during orthopedic surgery. As most patients are asymptomatic, treatment is usually not required, though calcium and vitamin D supplementation are commonly employed as part of management. When symptomatic patients experience bone pain or chronic back pain, bone deformities, and arthritis. Initial treatment is with NSAIDs and analgesics, if this is unsuccessful then bisphosphonates like risedronate can be used or antiresorptive therapy such as etidronate. Paget disease is associated with primary hyperparathyroidism and increased risk of primary bone neoplasms, especially osteosarcoma.


Set pelajaran terkait

18. Unit 6: Lesson 2: LS Assignment 3

View Set

Pg. 341 Larry Examples and Exercise #5

View Set

DHS Hazard Communication Standard

View Set

Principles of Real Estate II (Chapter 2 Vocabulary)

View Set

Tetracyclines, Aminoglycosides, & Others

View Set

Human Anatomy, CH 1, Test Prep Questions

View Set